Chem/Phys MQL

Pataasin ang iyong marka sa homework at exams ngayon gamit ang Quizwiz!

An acid with which of the following acid dissociation constants (pKa) would be the strongest? A) -1.0 B) 0.5 C) 3.0 D) 7.0

A; A strong acid will dissociate completely so the dissociation constant (Ka) will be higher for a strong acid compared to a weak acid. Since pX = -log X, the Ka is inversely proportional to the pKa . A strong acid will have the highest Ka and the lowest pKa ; therefore, choice (A) .

As oxygen is consumed underwater, the body also produces carbon dioxide. This has the effect of: A) decreasing the pH of the blood B) increasing the pH of the blood C) stabilizing the pH of the blood D) consuming plasma bicarbonate

A; An increase in carbon dioxide production in the body leads to more carbonic acid in the bloodstream, lowering the pH of the blood. Choice (A) is correct. Choices (B) and (C) are incorrect because they misidentify the relationship between CO2 and blood pH. choice (D) is incorrect because production of CO2 increases, not decreases, plasma bicarbonate levels.

In the first step of the reaction proposed in the passage, atomic oxygen is formed from carbon dioxide. Atomic oxygen is extremely reactive, and not found in any significant quantity on Earth's surface, because: A) it is a free radical B) it has a low effective nuclear charge C) it is a gas in its standard state D) it has 8 valence electrons

A; Atomic oxygen has 8 electrons, 2 inner and 6 valence with an electron configuration of 1s^2 2s^2 2p^4. Of the four electrons in the p orbital, 2 are paired in the -1 orbital, while 1 is unpaired in the 0 orbital and another is unpaired in the +1 orbital. These unpaired valence electrons, also known as free radicals, are extremely chemically reactive and explain the lack of [O] on earth's surface. Choice (A) is correct. Choice (B) is wrong because Oxygen actually has a relatively large effective nuclear charge (or Zeff), contributing to many of its properties, including its high electronegativity. Choice (C) is wrong because it does not answer the question, even though it is true (oxygen is a gas in its standard state); there are plenty of standard-state gas molecules that exist in abundance on earth. Choice (D) is wrong because oxygen only has 6 valence electrons; two of its total electrons are inner, non-valence electrons.

Charles' Law states that: A) at constant pressure, the volume of a gas is directly proportional to temperature B) at constant temperature, the volume of a gas is inversely proportional to pressure C) at constant temperature and pressure, the volume of a gas is directly proportional the number of moles of gas D) the total pressure exerted by a gaseous mixture is equal to the sum of the partial pressures exerted by each individual gas

A; Charles' law relates volume and temperature for a certain pressure and says that they are directly proportional V1/T1 = V2/T2 , or choice (A).

*See #31 on Kaplan Test #1* The action of phenylalanine hydroxylase on phenylalanine results in: A) conservation of stereochemistry, since the primary chiral center is not altered B) inversion of configuration, because the molecule is held within the active site C) racemization, because the -OH group is added to a planar intermediate D) formation of a meso -compound, because there is a plane of symmetry

A; Chiral centers are sp^3 hybridized, and attached to 4 different compounds. The carbon that is attached to the hydroxyl group in tyrosine is sp^2 hybridized in both phenylalanine and tyrosine, thus is not a chiral center. The chiral center in both molecules is the alpha carbon (relative to the carboxyl group). Since it is not changed in the reaction, stereochemistry is conserved, and choice (A) is correct. Choice (B) is wrong because while enzymes are typically stereospecific, they may act to conserve, invert, or racemize compounds. Being held within the active site is a feature of all three, and so inversion of configuration does not automatically follow from being within the active site. Choice (C) is wrong because racemization requires substitution at a chiral center. Choice (D) is wrong because meso compounds have an internal plane of symmetry for the entire molecule, but this compound only has a plane of symmetry through the phenyl group.

A diet soda is analyzed with high-pressure liquid chromatography, and saccharine is determined to be the first compound to reach the detector. Which of the following explains why saccharine reached the detector first? A) saccharine is the component of the diet soda that has the lowest affinity for the stationary phase being used B) saccharine is the component of the diet soda that has the highest affinity for the stationary phase being used C) saccharine is the component of the diet soda that has the lowest molecular weight D) saccharine is the component of the diet soda that has the greatest positive charge

A; Chromatography separates compounds based on their affinity for the stationary phase; the lower the affinity of the compound for the stationary phase, the faster it will be eluted through the column. This means that the compound that has the lowest affinity for the stationary phase will be the first compound to reach the detector, which is located at the end of the column. Choice (A) is correct.

What serves as the backbone for DNA? A) sugars linked by phosphodiester bonds to phosphate groups B) sugars linked by sulfodiester bonds to sulfate groups C) nitrogenous bases linked by phosphodiester bonds to phosphate groups D) nitrogenous bases linked by sulfodiester bonds to sulfate groups

A; DNA consists of a sugar-phosphate backbone that has nitrogenous bases attached to the sugar rings. This means that the backbone only consists of sugars and phosphates, or choice (A).

For sodium carbonate, Na2CO3 , what are the oxidation numbers of Na, C, and O, respectively? A) +1, +4, -2 B) -1,-4, +2 C) +2, +2, -2 D) -3, 0 , +2

A; For group IA elements, their oxidation number in a compound is usually +1. Therefore, looking at the first number in every answer the only correct one is (A) .

*Look at #21 from Kaplan Test #1* From the information provided in the passage, which of the following is necessarily true? A) Keq>1 B) delta Gf^o >0 C) the forward reaction is endothermic D) the potential energy of A is greater than the potential energy of D

A; For this question, evaluate each statement one by one, keeping the major points of the passage in mind as a prediction. Paragraph 3 states that the reaction proceeds spontaneously to the right; therefore the equilibrium constant must be greater than one. Thus, choice (A) is the correct answer. (B) is incorrect because if delta G were greater than zero, the reaction would not be spontaneous. (C) is incorrect because we don't have enough information to determine if the reaction is endothermic or exothermic. Free energy (as denoted by the y-axis in Figure 1), gives a measure of spontaneity, not enthalpy. Most spontaneous reactions are exothermic, but keep in mind that an endothermic spontaneous process is possible with a positive entropy change and a high temperature. So while it is possible that the forward reaction is endothermic, it is not necessarily true. Finally, (D) is incorrect because we cannot say anything about the potential energy of A alone or of D alone; we know only the potential energy of 2A+B is greater than that of 2C+D.

Which of the following would NOT affect the rate of a reaction? A) lowering the enthalpy of the reaction B) increasing the temperature C) lowering the activation energy of the reaction D) increasing the frequency at which the reactants collide

A; From the Arrhenius equation, the rate is dependent on the temperature, the activation energy, the gas constant, and the frequency factor. The equation takes the form: k = Ae^(- Ea/RT) Therefore the enthalpy of the reaction does not affect the rate and choice (A) is the correct answer.

If a five meter board is inclined to 30 degrees above a horizontal surface, how high above the surface is the highest point on the board? A) 2.5m B) 3m C) (5√(2))/2 m D) (5√(3))/2 m

A; Here the angle of the board is opposite the side that is being measured. This means that h x sin(30) = height. Since sin(30) = 1/2, the height should be 2.5 meters, or choice (A).

Captive mammals and birds that are forced to dive display an overall decrease in metabolism. As a result of such dives, the concentration of which of the following is likely to be elevated? A) lactate dehydrogenase B) pyruvate dehydrogenase C) alcohol dehydrogenase D) isocitrate dehydrogenase

A; In animals with reduced metabolism and reduced access to oxygen, anaerobic metabolic mechanisms will tend to dominate. In animals, anaerobic metabolism (fermentation) produces lactic acid, which is toxic. Therefore, such animals would be expected to have higher concentrations of enzymes that process lactic acid. Match to choice (A). Pyruvate dehydrogenase and isocitrate dehydrogenase both participate in aerobic cellular respiration, while alcohol dehydrogenase will break down ethanol (the product of fermentation in yeast).

After using the Sanger (chain-termination) method for DNA sequencing, a researcher utilizes gel electrophoresis and obtains a single dark band. Which of the following sources of error is most likely? A) Dideoxynucleotides were omitted from the reaction mixture B) The DNA was denatured into single strands using heat C) Deoxynucleotides were present in excess D) The primer annealed to the template strand in numerous places

A; In the absence of dideoxynucleotides, DNA polymerase will replicate the entire template and thus produce complete copies - all of the same length, so (A) is correct. (B) is a necessary step for replication to take place. (C) is inherent in gene sequencing and (D) would produce multiple products.

When an acyl halide reacts with a primary alcohol, which of the following will form? A) an ester B) an ether C) a secondary alcohol D) a carboxylic acid

A; In this reaction ,the carbon of the acyl halide acts as the electrophile; it is having electron density pulled away by both the carbonyl oxygen and the halogen. Alcohol, with its lone pairs of electrons on oxygen, can then attack, pushing up electrons onto the carbonyl oxygen. Those electrons reform a double bond kicking off the halide, which is a good leaving group. Lastly, the oxygen from the alcohol group loses its proton (and positive charge) and we are left with an ester, choice (A).

A lamp with a 100-watt light bulb is turned on. How long does it take for the lamp to consume 3.6x10^5 joules? A) 3.6x10^3 s B) 3.6x10^7 s C) (1/3.6)x10^-3 s D) (1/3.6)x10^-7 s

A; Remember that P = W/t. The watt is a unit for power, and is equal to 1 joule/sec. So, P=100 watts and W=3.6×10^5 joules. Rearranging the expression and solving for t, yields t = W/P = 3.6 × 10^5 J/100 W = 3.6 × 10^3 s, choice (A) .

In an experiment to determine the efficiency of a new assistive device for seniors, a kinesiologist sets up an experiment with several weights and pulleys. When he calibrates the scale it registers each weight as three kilograms greater than it actually is. What impact does this have on the experiment? A) the efficiency will be most skewed for the smallest weights B) the efficiency will be skewed equally for all weights C) the efficiency will be least skewed for the smallest weights D) the constant difference will not skew the efficiency measures

A; Since efficiency = (load x load distance) / (effort x effort distance), if the weights (load) were erroneously high but the effort was measured accurately, efficiency measures would be erroneously calculated as higher than actual efficiency. Hence (D) can be eliminated. Since efficiency is directly proportional to load, error in efficiency (%) is directly proportional to error in load (%). The error in load (%)= absolute error/actual load. As actual load decreases, the percent error increases. Hence (B) and (C) are incorrect and choice (A) is the correct answer.

100 mL of an ideal gas is placed in a sealed container at a pressure of 300 mmHg. If the pressure is increased to 450 mmHg, what is the final volume of gas? A) 67 mL B) 75 mL C) 100 mL D) 134 mL

A; Since only P and V are changing in this scenario, one must use Boyle's Law (P1 V1 = P2 V2 ) to determine the final volume of the gas. Plugging these values into the equation: P1 V1 = P2 V2 (300 mmHg)(100 mL) = (450 mmHg)(X mL) X = 67 mL which matches choice (A) .

What is the concentration of Ag+ in a saturated solution of AgCl at 25°C? (Ksp of AgCl = 1.8 × 10^-10 )? A) 1.34x10^-5 M B) 1.8x10^-10 M C) 3.2x10^-10 M D) 3.2x10^-20 M

A; The concentration or molar solubility of Ag + can be determined by writing an expression for the Ksp . The Ksp of AgCl can be written as Ksp = [Ag+ ] [Cl- ]. Assuming that the molar solubility of AgCl is X the Ksp can be simplified as X^2 . Since the Ksp of AgCl = 1.8 × 10^-10 , so X is equal to ~1.4 × 10^-5M . Since the molar ratio of AgCl to Ag+ is 1:1, the concentration of AgCl is also ~1.4 × 10^-5M . The closest number in the answer choices is choice (A) .

Which of the following best expresses the algebraic relationship between molar solubility, x, of Ca3(PO4)2 and its solubility product constant Ksp? A) Ksp= 108x^5 B) Ksp= 72x^5 C) Ksp= 32x^5 D) Ksp= 81x^5

A; The expression asked for can be derived by setting up the equilibrium expression for the dissolution of calcium phosphate, which will produce 3x moles of calcium ions and 2x moles of phosphate for each x moles of calcium phosphate dissolved. Algebraically, we can rearrange it to solve for Ksp in terms of molar solubility: Ca3(PO4)2 -> 3 Ca ^2+ + 2 PO4^3- Ksp= [Ca^2+]^3 [PO4^3-]^2 = (3x)^3 (2x)^2 = (27x^3)(4x^2)= 108x^5 This matches choice (A).

A conservationist would like to test the acidity of a sample of rainwater by titrating it with 0.05M NaOH. What additional information is needed to calculate the initial number of moles of H2CO3 in the sample? I. volume of NaOH used to reach the end point II. Ka of H2CO3 III. Initial volume of rainwater in the sample A) I only B) I and II only C) I and III only D) I, II, and III

A; The goal is to find the number of moles of H2CO3, not the concentration. Therefore, it is only necessary to know how many moles of NaOH were required to reach equivalence, which is Roman numeral I. Regarding Roman numeral III, it does not matter what volume of rainwater we start with if we are only concerned with the number of moles, since each mole of OH- added into solution will react with one mole of H+ from the weak acid - the only thing volume would tell us is the concentration of the starting material once we've found the initial moles. Finally, the Ka of the weak acid does not matter, as we are titrating with a strong base. Once OH- is introduced into solution, it will remove the H+ no matter how strong or weak the acid, so Roman numeral II is not relevant. Since statement I is true but statements II and III are note, (A) is correct.

500 calories of heat are applied to a 10g cube of ice that is at an initial temperature of 0 °C. What is the change in temperature of the H2O due to this amount of heat? (Note: the heat of fusion of water is 80 cal/g and the specific heat of water is 1 cal/g*°C) A) 0 °C B) 5/8 °C C) 42 °C D) 50 °C

A; The ice cube is in a solid state, and requires heat of fusion for it to melt into liquid water. Its temperature will not increase until it is in the liquid state. As given in the question stem, the heat of fusion of water is 80 cal/g. Since we have a 10 g ice cube, we would theoretically need 800 cal in order to simply melt the ice cube. The question stem states that only 500 cal is being applied — that's not enough to melt the ice cube, let alone have extra energy to increase the temperature of the water. Therefore choice (A) , 0 °C, is correct.

The force of friction acting on an object is 50N, and the net force on that object is also 50N. The object is: A) accelerating B) motionless C) moving at a constant velocity D) not enough information to determine the state of motion

A; The question stem states that there is a net force acting on the object, therefore, by Newton's Second Law, the object is accelerating. Since there is a net force of 50 N and a frictional force of 50 N, there must be 100 N of force opposing friction.

*See #35 on Kaplan Test #1* Based on the results of the experiment, which of the following conclusions can be made about the reaction in Figure 1? I. PAH increases the reaction rate II. PAH reduces the delta H between the reactants and products III. the reaction rate peaks at 30 degrees C A) I only B) II only C) I and II only D) I and III only

A; The reaction rates in Trial 1 are higher than in Trial 2 (notice that Trial 2's numbers are in pM, which is 10^3 less concentrated than nM), so PAH increases the reaction rate. Statement I is true. Enzymes only affect the kinetics of a reaction, not its thermodynamics. Delta H (enthalpy change) is a thermodynamic measure, and is unaffected by PAH. So statment II is false. While the reaction rates are shown to increase with temperature, we do not have enough data to suggest that the rates peak at 30 degrees C. Rates at even higher temperatures would need to be measured in order to check the validity of statement III. And in fact, we should suspect that rates will continue to increase with temperature, since body temperature of 37 degrees C. Since statement I is a valid conclusion and statements II and III are not, choice (A) is correct.

*See figure for question #8 on Kaplan Test 2 (short)* An unknown compound was found in the lactated Ringer's solution. The IR spectrum of the compound is shown below. The functional group indicated by the arrow in the figure below is a: A) carbonyl group B) alcohol group C) methyl group D) amino group

A; The sharp peak at 1700 cm^-1 is indicative of the carbonyl functional group. Choice (A) is correct.

Which of the following is a false statement about the reaction below? CH3Ch2CH2Cl --(NaOH/DMSO)--> CH3CH2CH2OH A) the reaction proceeds via the formation of a carbocation B) the rate of reaction is dependent upon the concentration of both substrate and nucleophile C) the reaction proceeds via a concerted mechanism D) the reaction intermediate is neutral

A; The solvent DMSO is a polar aprotic solvent and the substrate is a primary alkyl halide, so this reaction will proceed through an S N 2 mechanism. The formation of a carbocation intermediate is a property of S N 1 reactions. Choice (B) . S N 2 reactions are bimolecular reactions so the rate of a S N 2 reaction will increase when the concentration of either the substrate or nucleophile is increased. Choice (C) . S N 2 reactions proceed through a concerted reaction where the nucleophile will add to the carbon skeleton of the electrophile during the same moment that the leaving group is removed from the electrophile. The reaction therefore occurs in one step with the formation of one transition state. Choice (D) . Compared to a carbocation that is formed in S N 1 reaction, which is positively charged, the transition state formed has neither a positive nor a negative charge.

Nitrophenols are highly toxic for humans, and are often found contaminating the soil near former factories. Which of the following statements about nitrophenols provides the most likely explanation for the data in the table below? Isomer Solubility (g/dL) Melting Point (C) 1-hydroxy-4-nitrobenzene 1.7 114 1-hydroxy-3-nitrobenzene 1.4 97 1-hydroxy-2-nitrobenzene 0.2 44 A) 1-hydroxy-2-nitrobenzene has reduced intermolecular bonding due to intramolecular bonding B) 1-hydroxy-4-nitrobenzene has the weakest intermolecular hydrogen bonding C) 1-hydroxy-3-nitrobenzene forms intramolecular hydrogen bonds D) 1-hydroxy-2-nitrobenzene does not form intermolecular bonds

A; These compounds have two functional groups that can hydrogen bond: the nitro (NO2) group and the hydroxyl (OH) group. Any two of these functional groups that are in close enough proximity will hydrogen bond, whether they are on different molecules or on the same molecule. How they align depends upon the shape of the molecule, which is different for all three. Furthermore, the melting and boiling points and water solubilities of polar compounds are largely governed by the extent of intermolecular hydrogen bonding. According to the table, 1-hydroxy-4-nitrobenzene and 1-hydroxy-3-nitrobenzene are more water-soluble and have higher melting points than 1-hydroxy-2-nitrobenzene. This is because 1-hydroxy-2-nitrobenzene tends to form inTRAmolecular hydrogen bonds instead of inTERmolecular bonds; since the NO2 and OH group are on adjacent carbons, they are able to hydrogen bond with each other. That makes the molecules less likely to form intermolecular bonds, so that it takes less energy to make them separate. Thus, they will melt at a relatively low temperature. Likewise, intramolecular hydrogen bonding reduces the molecule's ability to hydrogen bond with water, making the compound less water-soluble. therefore, (A) is correct. (B) is incorrect because 1-hydroxy-3-nitrobenzene and 1-hydroxy-4-nitrobenzene form strong inTERmolecular hydrogen bonds - that's why they have higher melting points. Also, since form more hydrogen bonds with water, they are more water-soluble. (C) is incorrect because the hydroxyl and nitro groups in 1-hydroxy-3-nitrobenzene are too far apart to hydrogen bond with each other (they are not on adjacent carbons). Finally (D) is incorrect because 1-hydroxy-2-nitrobenzene will form SOME intermolecular hydrogen bonds, just not as many as the other two isomers.

*See passage for #21 on AAMC Test 2* Which statement about the cooperativity of RIα/C activation and RIα protein folding is supported by the data in figures 2 and 3? A) both activation and folding are cooperative B) activation is cooperative, but folding is not C) folding is cooperative, but activation is not D) neither activation nor folding is cooperative

A; This is a Biochemistry question that falls under the content category "Principles of chemical thermodynamics and kinetics." The answer to this question is A because both curves have a sigmoidal shape, which is indicative of cooperative processes. It is a Data-based and Statistical Reasoning question because you must interpret graphical representations of data to determine the properties of a protein.

Protein secondary structure is characterized by the pattern of hydrogen bonds between: A) backbone amide protons and carbonyl oxygens B) backbone amide protons and side chain carbonyl oxygens C) side chain hydroxyl groups and backbone carbonyl oxygens D) side chain amide protons and backbone carbonyl oxygens

A; This is a Biochemistry question that falls under the content category "Structure, function, and reactivity of biologically-relevant molecules." The answer to this question is A because secondary structure is represented by repeated patterns of hydrogen bonds between the backbone amide protons and carbonyl oxygen atoms. It is a Knowledge of Scientific Concepts and Principles question because you must work with the scientific theory of secondary structure to determine which components of a protein are indicative of that structure.

*See passage for #23 on AAMC Test 2* Based on the data presented in figures 2 and 3, what is the most likely role of Y229 in protein stability and cAMP activation? A) Y229 is important for protein stability but not critical for cAMP activation. B) Y229 is important for cAMP activation but not critical for protein stability. C) Y229 is important for protein stability and critical for cAMP activation. D) Y229 is not important for protein stability and not critical for cAMP activation.

A; This is a Biochemistry question that falls under the content category "Structure, function, and reactivity of biologically-relevant molecules." The answer to this question is A because the thermal melt shows that removal of Y229 decreases stability, therefore Y229 is important for stability. Removal of Y229 has little effect on protein activation, as the activation curve is similar to WT activation. It is a Data-based and Statistical Reasoning question because you must reason with the melt curve data and the activation data to determine the role of a specific amino acid in the protein.

*See passage for #38 on AAMC Test 3* Which structure represents a component of the HRP cofactor? A) *see image B) *see image C) *see image D) *see image

A; This is a Biochemistry question that falls under the content category "Structure, function, and reactivity of biologically-relevant molecules." The answer to this question is A, because the cofactor is heme, which is a porphyrin. The basic unit of a porphyrin is the pyrrole ring, a five-sided heterocycle containing one nitrogen atom. It is a Knowledge of Scientific Concepts and Principles question because recalling that heme is a porphyrin and remembering the structure of the ring components of a porphyrin are essential for answering this question.

*See passage for #55 on AAMC Test 3* If the reaction shown in Equation 1 is spontaneous, the value of K must be: A) greater than 1 B) equal to ΔG C) less than 1 D) equal to zero

A; This is a General Chemistry question that falls under the content category "Principles of chemical thermodynamics and kinetics." The answer to this question is A, because a spontaneous reaction is one that exhibits ΔG < 0. Since ΔG = -RTln(Keq), this means that Keq must be > 1. It is a Knowledge of Scientific Concepts and Principles question because you must identify the relationship between the closely related concepts of free energy and equilibrium constant for a reaction.

*See passage for #42 on AAMC Test 1* The radioactive decay described in the passage results in the formation of which two atoms? A) N and S B) B and Si C) B and S D) N and Si

A; This is a Physics question that falls under the content category "Atoms, nuclear decay, electronic structure, and atomic chemical behavior." The answer to this question is A because the type of radioactive decay is b-1 so a neutron is converted into a proton, which results in carbon becoming nitrogen and phosphorus becoming sulfur. It is a Scientific Reasoning and Problem Solving question because you are asked to reason about the scientific theory that correlates transmutation of elements and nuclear radioactivity.

Which of the following properties of a 2.3 MHz ultrasound wave remains unchanged as it passes into human tissues? A) Frequency B) Wave speed C) Amplitude D) Wavelength

A; This is a Physics question that falls under the content category "How light and sound interact with matter." The answer to this question is A because the frequency of a wave is not affected by the medium through which it propagates. It is a Knowledge of Scientific Concepts and Principles question because it asks you to use data to recognize correct scientific principles.

Which measurement unit CANNOT be used to express power? A) kg•m^2 •s^2 B) J•s^-1 C) ft•lb•s^-1 D) W

A; This is a Physics question that falls under the content category "Translational motion, forces, work, energy, and equilibrium in living systems." The answer to this question is A because the measurement unit of power is watt, defined as J/s = ft•lb/s = kg•m^2/s^3. It is a Knowledge of Scientific Concepts and Principles question because you are asked to identify a relationship between closely related concepts.

*See passage for #53 on AAMC Test 2* How much work did an 83-year-old female do while stretching the rubber band to the limit of her strength? A) 4 J B) 5 J C) 6 J D) 7 J

A; This is a Physics question that falls under the content category "Translational motion, forces, work, energy, and equilibrium in living systems." The answer to this question is A because the work done is W = 0.5 × kx^2 where x = 0.20 m. So W = 0.5 × 200 (N/m) × (0.2 m)^2 = 4.0 J. It is a Scientific Reasoning and Problem Solving question because the question requires determining and using scientific formulas to solve problems.

*See image for #3 on AAMC Test 1* According to the developed chromatography plate shown below, what is the approximate Rf value of aspartic acid? A) 0.20 B) 0.50 C) 5 D) 10

A; This is an Organic Chemistry question that falls under the content category "Separation and purification methods." The answer to this question is A (0.20) because Rf is the ratio of the distance travelled by the analyte relative to the solvent front during a chromatographic separation. Aspartic acid travelled two units, while the solvent front travelled ten units, giving an Rf of 2/10 = 0.20 for aspartic acid. It is a Data-based and Statistical Reasoning question because you are asked to analyze and interpret data presented in a figure to draw a conclusion.

Four organic compounds: 2-butanone, n-pentane, propanoic acid, and n-butanol, present as a mixture, are separated by column chromatography using silica gel with benzene as the eluent. What is the expected order of elution of these four organic compounds from first to last? A) n-Pentane → 2-butanone → n-butanol → propanoic acid B) n-Pentane → n-butanol → 2-butanone → propanoic acid C) Propanoic acid → n-butanol → 2-butanone → n-pentane D) Propanoic acid → 2-butanone → n-butanol → n-pentane

A; This is an Organic Chemistry question that falls under the content category "Separation and purification methods." The answer to this question is A. The four compounds have comparable molecular weights, so the order of elution will depend on the polarity of the molecule. Since silica gel serves as the stationary phase for the experiment, increasing the polarity of the eluting molecule will increase its affinity for the stationary phase and increase the elution time (decreased Rf). It is a Reasoning about the Design and Execution of Research question because you must reason about the features of a research design to identify a causal relationship.

*See passage for #16 on Kaplan Test 2 (short)* Sucrose does NOT produce a positive test when treated with Benedict's solution because: A) the carbonyl functionalities of both subunits are involved in the formation of a glycosidic linkage B) it undergoes mutarotation C) the molecule is not oriented correctly for a reaction to occur D) disaccharides do not possess free hemiacetal or hemiketal groups

A; To answer this question, it is necessary to know that Benedict's reagent is a common reagent used to test for reducing sugars; in other words, those sugars that contain hemiacetal or hemiketal groups. Only sugars with free carbonyl groups (i.e. hemiacetals or hemiketals) can undergo oxidation and so reduce Benedict's, producing a positive result for the test. Anyway, back to the question. Sucrose doesn't give a positive Benedict's test since it doesn't possess any free carbonyl groups. Instead, these groups are tied up in forming the glycosidic bond between the two monosaccharide subunits, so choice (A) is the correct answer. Choice (B) is incorrect as sucrose will not undergo mutarotation. As stated earlier, sucrose doesn't possess any free hemiacetal or hemiketal groups and the fact that it doesn't undergo mutarotation is a consequence of this. Choice (C) is wrong, as the orientation of the molecule has nothing to do with the fact that it will not reduce Benedict's. Finally, choice (D) is wrong because some disaccharides DO possess free hemiacetal and hemiketal groups. For example, maltose consists of two glucose residues, one of which is present in the hemiacetal form.

What is the delta H for Reaction 1? (delta Hf for CO2 = -393.5 kJ/mol delta Hf for H2O= -285.8 kJ/mol delta Hf for C6H12O6= -1274.5 kJ/mol) A) +2801.3 kJ/mol B) -2801.3 kJ/mol C) +5350.3 kJ/mol D) -5350.3 kJ/mol

A; To solve this problem, use the formula for delta H of a reaction: H= Hf(products)-Hf(reactants) Plugging in the appropriate values from Table 1: H= [Hf(C6H12O6)+6Hf(O2)]-[6Hf(CO2)+6Hf(H2O)] = [-1300+0]-[6(-400)+6(-300)] = -1300 - (-4200) = 2900 This prediction is closest to +2801.3, or choice (A).

*Look at #23 on Kaplan Test #1* A researcher using Method 1 to determine the capillary pressure fills the pipette with an inert fluid less dense than blood. Compared to blood, the height of this fluid in the pipette will be: A) higher because the fluid is less dense B) lower because the fluid is less dense C) the same because the pressure being measured is the same D) the same because the velocity of blood flow in the capillary bed is the same

A; the passage states that using Method 1, the capillary pressure is given by P= (rho)gh. We are measuring the same capillary, so the measured pressure, P, is constant. Gravity, g, is also constant here. So if we decrease rho (signifying density) by using a less dense fluid, h will have to increase to compensate for it and keep P equal. Choice (A) is correct. Choice (D) is a choice that may have been tempting if you remembered that Bernoulli's equation relates the velocity of fluid flow to the pressure. However, an equal pressure applied to a less dense fluid will cause it to rise more, not stay at the same level as a more dense fluid.

*See passage for #20 on Kaplan Test 2 (short)* Sucrose can also be referred to by which of the following names? I. alpha-D-Glucopyranosyl beta-D-fructofuranoside II. alpha-D-Glucofuranosyl beta-D-fructopyranoside III. beta-D-fructofuranosyl alpha-D-Glucopyranoside IV. alpha-D-fructofuranosyl beta-D-Glucopyranoside A) I only B) I and III only C) II and III only D) II and IV only

B; A common way to name cyclic monosaccharides is according to the ring size: "pyran" corresponds to a 6-membered ring, while "furan" corresponds to a 5-membered ring. It's logical then that a five-membered fructose ring would be a fructofuranose, while a 6-membered glucose ring would be a glucopyranose. However, since the subunits in sucrose are glycosides, the acetal forms of glucose and fructose would be glucopyranoside and fructofuranoside. Therefore, Roman numeral II can be ruled out immediately, since it states that glucose will be a furanoside, not a pyranoside, and that fructose will be a pyranoside, not a furanoside. Choices (C) and (D) can thus be eliminated. Furthermore, the suffix for a glycoside is "-osyl", (for example, fructofuranoside is fructofuranosyl). Sucrose can therefore be named as a fructofuranosyl-glucopyranoside or a glucopyranosyl-fructofuranoside; Roman numerals I, III, and IV are possible answer choices. The last thing to look at is the alpha and beta designations. The glucoside has an alpha linkage, while the fructoside has a beta linkage. You should be able to see that choice IV is wrong, as this describes the fructofuranoside as alpha, while the glucopyranoside is beta. As sucrose can be named as a fructofuranoside or a glucopyranoside, statements I and III are equally viable. Statements I and II both have the proper ring size and anomeric designations. Therefore, choice (B) is the correct answer.

An electron with a speed of 3 × 10^5 cm/s moves in a northerly direction through a region of space containing a uniform electric field of strength 100 V/m, directed toward the east. Which of the following best describes the effect of the field on the motion of the electron? A) the electron is deflected to the east with no change in speed B) the electron is deflected to the west and experiences an increase in speed C) the electron is deflected to the east and experiences an increase in speed D) the electron is deflected to the west with no change in speed

B; A negative charge in an electric field feels a force in the direction opposite the field. Since the field points toward the east, then, the electron should feel a force toward the west. Thus, eliminate choices (A) and (C) . And, according to Newton's second law, a net force applied to a body will result in the body undergoing acceleration in the same direction as the net force. So, the acceleration associated with the electric force will act to increase the velocity in the westward direction. Since the electron did not already have a component of velocity in either the westward or eastward direction, this will have the effect of increasing the total speed of the electron. This makes choice (B) correct.

Citric acid is a weak triprotic acid. Because it has multiple acidic protons, a solution of citric acid can: A) not act as an effective buffer at a pH near its pKa2 value B) act as a buffer over several pH ranges C) only act as an effective buffer at a pH near its pKa2 value D) not act as an effective buffer at a pH near any of its pKa values

B; All the answer choices involve citric acid's functionality as a pH buffer. A buffer resists a change in pH. This relationship can be described using the Henderson-Hasselbalch equation where pH varies least around the pKa of a buffer when there is an equal proportion of weak acid and conjugate base. Because citric acid is triprotic, it has 3 pKa values and 3 different weak acid states (H3A, H2A -, and HA 2-). This gives it the ability to act as a buffer across three different pH ranges, which in the case of citric acid overlap one another slightly. Choice (B) is correct.

Which of the following ketones will have the most acidic alpha-hydrogen? A) (H3C)-(CO)-(CH3) B) (Cl3C)-(CO)-(CH3) C) (H2NH2C)-(CO)-(CH3) D) (Cl3C)-(CO)-(CCl3)

B; Alpha hydrogens are relatively acidic because of resonance stabilization as shown below. The acidity of the alpha hydrogens will be increased by proximity to electron withdrawing groups. Since the chloride groups in choice (B) are electron-withdrawing, they will help stabilize the negative charge, and so choice (B) is the correct answer. As for the incorrect answer choices, (A) is incorrect because dimethylketone has no additional electron withdrawing groups; (C) is incorrect because an amine is an electron-donating group and would tend to decrease the acidity of the alpha hydrogen; (D) is incorrect because there is no alpha hydrogen to be removed.

A diabetic patient displays acidosis due to accumulation of ketone bodies in the blood. What compensatory changes would take place in the body? A) breathing rate would increase and total blood CO2/HCO3 - concentration would increase B) breathing rate would increase and total blood CO2/HCO3 - concentration would decrease C) breathing rate would decrease and total blood CO2/HCO3 - concentration would increase D) breathing rate would decrease and total blood CO2/HCO3 - concentration would decrease

B; As Paragraph 3 states, the immediate buffering effect of bicarbonate is controlled by changes in the breathing rate. When acidosis occurs, the concentration of H+ is too high. Le Châtelier's principle applies: in order to decrease the concentration of H+, the concentration of carbon dioxide must decrease. If the breathing rate increases, more carbon dioxide is exhaled and its concentration in the blood decreases. Since the effect of this rapid breathing is to remove carbon dioxide from the body, the ultimate effect is to decrease the total CO2/HCO3- concentration in the blood. Therefore, choice (B) is correct.

*See graphs for #47 on Kaplan Test 2 (short)* As blood travels through the pulmonary and systemic circuits it experiences a change in gravitational potential energy as measured with reference to the heart. Which graph best matches this change during a single loop through the pulmonary and systemic circuits? A) *see graph B) *see graph C) *see graph D) *see graph

B; As blood moves from the heart to the lungs, it moves to a slightly greater height, leading to a small gain in gravitational potential energy. This potential energy falls back to earlier levels as the blood returns from the lungs to the heart. Thus the pulmonary circuit would correspond to a small bump on the graph. Next, as the blood leaves the aorta, it initially moves higher, and then starts to fall rapidly till potential energy reaches its lowest point. As the venous blood starts to return to the heart, potential energy returns to baseline levels. The entire systemic circuit would thus correspond to a small bump-steep drop-steep climb back . The steep climb back will have a smaller slope than the steep drop, since venous flow is slower than arterial flow. Choice (B) is the only choice that matches this prediction, and is the correct answer choice.

Which compound is oxidized in Reaction 1? 6 CO2 + 6 H2O + light -> C6H12O6 + 6 O2 A) CO2 B) H2O C) C6H12O6 D) O2

B; As stated in Paragraph 2 ("These proteins use photons of certain energy levels to split water into hydrogen and oxygen, abstracting electrons from the hydrogen atoms and producing NADPH and ATP."), electrons are taken from the hydrogen of the water, so water must be oxidized. The correct answer is (B).

Dividing the conservation of energy equation by which value gives Bernoulli's equation? A) area B) volume C) distance D) work

B; Bernoulli's equation takes conservation of energy and divides by volume to give a relationship between pressure, velocity, and height. This matches choice (B) .

What is the order of increasing boiling points in the following compounds? I. CH3CH2COOH II. CH3CH2CHO III.CH3CH2CH2OH A) I < III < II B) II < III < I C) III < II < I D) I < II < III

B; Boiling points are a physical property of a compound that is determined solely by the strength of intermolecular forces. Intermolecular forces are the bonding interactions between two compounds unlike intramolecular forces that are within one compound. The greater the intermolecular forces between compounds, the greater the amount of energy that must be used to separate these compounds in the solid or liquid phase so that they are liberated as a gas. This energy is usually supplied in the form of heat, so a compound with a high boiling point will require more heat energy. Identify the functional groups of each of the choices and determine possible intermolecular forces that can result between two of the same compounds. Compound I is a carboxylic acid that can form two hydrogen bonds to another carboxylic acid. Compound II is an aldehyde that has a polar carbonyl group that can form dipole - dipole interactions. Compound III is a primary alcohol that can form one hydrogen bond to another primary alcohol. Therefore, the strength of the intermolecular forces and the boiling points in increasing order is II < III < I. Choice (A) . The question asks for the boiling points of the compounds in increasing order. The order is reversed here. Choice (C) . Hydrogen bonding is a stronger intermolecular force than a dipole - dipole interaction. Thus, the alcohol will be expected to have a stronger boiling point than the aldehyde. Choice (D) . Carboxylic acids can form dimers as a result of hydrogen bonding and will require the most amount of heat energy to separate them. This will translate to a very high boiling point.

Which of the following electrochemical conditions will result in the greatest amount of copper plated from a copper (II) sulfate solution? A) a current of 10 amperes in 5 minutes B) a current of 10 amperes in 15 minutes C) a current of 5 amperes in 10 minutes D) a current of 5 amperes in 15 minutes

B; Copper will be plated when the copper (II) ions in solution are reduced to copper metal on the electrode. The source of these electrons comes from the current, so more copper ions will be plated when the current is maximized. Current is defined as charge over a unit of time (q/t) so multiplying the current by a unit of time t will provide the amount of charge transferred to the copper ions. Maximizing the amount of time transferred will maximize the amount of copper plated. This can all be summarized using the equation I * t = F * n where F is the Faraday constant and n is the moles of metal plated. Therefore, the most amount of current for the longest time is desired and matches choice (B) .

*See passage for #53 on Kaplan Test #1* An experimenter working with the apparatus shown in Figure 1 uses a voltmeter and calculates the potential difference between points d and f as V. If the resistance of R2 = 1/R1 then what is the current (I) through point e? A) I=V/R1 B) I= VR1/(R1^2 +1) C) I= VR1 + V/R1 D) I = V/2R1

B; First it is necessary to realize that we are given the potential drop from points d to f and asked about the current through e. Since d, e, and f are in series, the current through each is the same. We are given the voltage drop from d to f and the resistance between, so it's a matter of rearranging and applying Ohm's Law: I=V/(R1 + R2). Substituting for R2, I= V/(R1 + 1/R1). Simplifying the fraction: I= V/[(R1)^2 /R1 + 1/R1]= V/[(R1)^2 + 1]/R1 = VR1/[(R1)^2 + 1], which matches (B). (A) would be correct if the given potential drop (V) was from d to e. (C) can be obtained by incorrectly simplifying the fraction: V/(R1 + 1/R1) does not equal VR1 + V/R1. (D) can be obtained by not substituting for R2.

Experimenters isolate a biological sample believed to be composed entirely of glycine. In order to confirm their belief, they can hydrolyze the peptide and then: A) use separation techniques to determine if the sample contains amino acids with hydrophobic R groups B) use polarized light in order to determine whether or not the sample is optically active. C) raise the pH of the sample and apply an electric field to test for the presence of zwitterions D) titrate the sample with NaOH to determine if it contains amino acids with acidic R groups

B; Glycine is the only amino acid that has no chiral center, as its R-group is simply a hydrogen atom. Therefore, a sample of pure glycine will not be optically active. The method described in choice (B) would help determine whether the protein contains any amino acids apart from gylcine, because as the passage states, most biological systems use only L-amino acids. Choice (A) is wrong because Gly would not have hydrophobic R groups, but neither would many other amino acids. Choice (C) is wrong because Gly, along with all other neutral amino acids, are zwitterions, and so there would be no difference to test. Also, the procedure described is an incomplete test for zwitterions. Choice (D) is wrong because Gly would not have an acidic R-group, but neither would many other amino acids.

*Look at passage for question #4 on Kaplan Test #1* Which of the following will increase the yield of carboxylic acid formed from a nitrile group-containing compound, using a mechanism similar to the Strecker synthesis? A) use a stronger nucleophile, like a Grignard Reagent, instead of H2O B) attach an electron withdrawing group to the carbon of the nitrile group C) use a stronger electrophile, like a Grignard Reagent, instead of H2O D) attach an electron donating group to the carbon of the nitrile group

B; In Step 6 we see a nitrile group (-CN) reacting with 2H2O in order to form a carboxylic acid. Even without knowing the mechanism beforehand, we know that H2O has lone pair electrons, and based on the passage, our products have new C-O bonds and no C-N bonds anymore. In fact, H2O acts as a nucleophile and the carbon in -CN acts as an electrophile (as it has its electron density drawn away by the N atom). If we attach another electron-withdrawing group to the carbon, as choice (B) suggests, that the carbon will be even more electron deficient, and act as a better electrophile, increasing the yield of carboxylic acid. Choice (A) is wrong because a Grignard Reagent is a stronger nucleophile than H2O but remember the goal is to make carboxylic acid, not new C-C bonds, which would be the product of a reaction with Grignard Reagents. Choice (C) is wrong because neither Grignard Reagents nor H2O are good electrophiles, and that would not help the reaction anyway. Choice (D) is wrong because an electron donating group would hinder this reaction for the exact opposite reason as described above.

Which of the following is the correct electronic configuration for radioactive iodine, an isotope used in thyroid imaging? A) [Kr]4d^10 5s^1 5p^6 B) Kr]4d^10 5s^2 5p^5 C) Kr]4d^10 5s^2 5p^4 D) Kr]4d^10 5s^2 5p^6

B; Isotopes differ in their number of neutrons, not electrons, so we are simply looking for the electronic configuration of iodine. (C) and (D) can be eliminated because they have an incorrect number of electrons (iodine has 53). And since iodine is not a transition element, it is not an exception to the filling order; therefore, the 5s shell will fill with 2 electrons before the 5p shell fills, and (B) is correct.

What is the product of reacting a ketone with LiAlH4? A) a primary alcohol B) a secondary alcohol C) a carboxylic acid D) an aldehyde

B; Lithium aluminum hydride, LiAlH 4 , is a reducing agent and will force carbonyls to become alcohols. Since the question stem indicates the reduction of a ketone, the carbonyl carbon is already attached to two other carbons. Reduction doesn't break down C-C bonds and so these will stay intact, resulting in (B) , a secondary alcohol.

*See passage for #50 on Kaplan Test #1* Even at a constant depth and temperature, a shift occurs inducing a net upward force. Assuming no changes are made to the buoyancy compensator, this is best explained because: A) the diver-equipment system occupies less volume over time B) he consumes more air and the exhaled vapor no longer contributes to his weight C) the air in the neoprene suit is replaced by water C) the density of the air in his lungs decreases, which decreases his specific gravity

B; Since the diver is remaining relatively still in relation to the earth, the only changes in gravitational force will result from changes in mass; for gravitational force to decrease, mass must decrease as well. Choice (B) is the only choice that fits this explanation, and it explains that air from the diver's tank is being consumed and discarded during the dive. Choice (A) may be true, but would lead to negative buoyancy. therefore it does not explain the observation at hand. Choice (C) is wrong because replacing air with water would increase the density of the suit. Choice (D) is wrong because the density of the air in the tank decreases as it is used, however the air in the lungs will always be at ambient pressure.

What is the first step in the Strecker synthesis of amino acids? A) nucleophilic attack on a nitrile carbon B) aminonitrile formation C) reaction of alkyl halide into a primary amine D) electrophilic attack by ammonia

B; Strecker synthesis is a two-step process to form amino acids. The first step is aminonitrile formation followed by protonation of the aminonitrile and attack by water to turn the nitrile into a carboxylic acid. This means the first step matches choice (B) .

Which of the compounds below cannot be oxidized to a carboxylic acid? A) an aldehyde B) a tertiary alcohol C) a primary alcohol D) a terminal alkene

B; Tertiary alcohols are not normally oxidized since a C-C bond would have to be broken to form a C-O bond. C-C bonds are very strong and stable, and thus resistant to an oxidizing agent. Choice (A) . An aldehyde can be easily oxidized into a carboxylic acid. Aldhedyes are one of the common reactants to produce carboxylic acids by oxidation. Choice (C) . A primary alcohol can be easily oxidized into a carboxylic acid. Primary alcohol are the only alcohols that can be used to produce carboxylic acids by oxidation since oxidized secondary alcohols form ketones that cannot be oxidized further Choice (D) . A terminal alkene can be oxidized into a primary alcohol, so it is a possible substrate to form a carboxylic acid using an oxidizing agent.

An ideal gas in a closed system undergoes a process in which it does 20 joules of work. If, after the process, the internal energy of the gas is 10 joules greater than it was before, then the amount of heat gained or lost by the gas during the process is: A) 10 joules lost B) 30 joules gained C) 10 joules gained D) 30 joules lost

B; The First Law of Thermodynamics says ΔU = Q - W, where ΔU is the change in internal energy, Q is the heat energy transferred to the body and W is the work done by the system. This question asks about Q, which is equal to ΔU + W. The question stem indicates the system does 20 joules of work. Thus, W = 20. We are also told that the gas experiences an increase in internal energy of 10 joules. Thus, ΔU = 10. Substituting these values into Q = ΔU + W, gives Q = 10 J + 20 J = 30 joules. The fact that this answer is positive and not negative indicates that these joules were gained, not lost, by the system, or choice (B) .

*See passage for #49 on Kaplan Test #1* If the neoprene suit occupied 4 liters on land, approximately how many moles of air would need to be added to the buoyancy compensator to maintain neutral buoyancy at a gauge pressure of 1 atm? A) 3.4/22.4 B) 1.6/22.4 C) 11.2 D) 22.4

B; The air in the neoprene initially occupied 1.6 L (40% of 4 L). The new pressure is 2 atm (remember, total pressure is atmospheric pressure plus gauge pressure). Using Boyle's law, we know that the new volume of air in the suit is 0.8 L. Therefore, we require 0.8 L of air from the buoyancy compensator, which would be 0.8/22.4 moles at 1 atm of pressure. Since the pressure is twice as high (2 atm) we require twice as many moles of gas, or 1.6/22.4. Choice (B) is therefore the correct answer.

Suppose that the liquid used in a thermometer has a volume expansion coefficient that is less than that of the thermometer tube itself. Which of the following best describes what happens with a change in temperature? A) the liquid level rises when the temperature rises B) the liquid level falls when the temperature rises C) the liquid level remains the same when the temperature rises D) the liquid level falls when the temperature falls

B; The amount of expansion, known as volume expansion, is proportional to the volume of the substance and the increase in temperature: ΔV = βVΔT, where ΔV is the change in volume, β is the coefficient of volume expansion (a property of the material), V is the original volume, and ΔT is the change in temperature. The question states that the liquid in the thermometer has a lower volume expansion coefficient than the thermometer tube itself. This means that, given the same amount of temperature increase, the liquid inside the tube will not expand as much as the tube itself. When the temperature is increased, the tube will be expanding and the liquid level will appear to be dropping because the tube is expanding faster than the liquid, or choice (B) .

The compound below is best classified as a(n): H2N-CH2-CO-NH-CH2COOH A) amino acid B) dipeptide C) monoglyceride D) carbohydrate

B; The compound contains one peptide bond between the carbonyl group of one amino acid and the amino end of another, so it is a dipeptide (consisting of two glycine residues). The peptide bond is formed between two amino acids where the -OH from the carboxyl end of one amino acid and a hydrogen from the second amino end are lost in the form of water. The peptide bond is an amide linkage and has a planar geometry. Choice (A) . An amino acid has a carbon attached to an amine, a carboxyl group, a hydrogen and a side chain. The compound referenced in the question stem has two amino groups and two carbonyl groups, so it must be more than one amino acid. Choice (C) . A mono-glyceride is a triglycerol molecule with a single fatty acid attached. Choice (D) . Carbohydrates are either aldehydes (aldoses) or ketones (ketoses) with many hydroxyl groups attached to them. Thus, they are not amino acids.

*See figure for #7 on Kaplan Test 2 (short)* In an attempt to understand the properties of arterial blood flow, scientists measure the volume of fluid displaced along the artery in Figure 1 as as function of time. How do the volume displacements per second at points A, B, and C compare to each other? A) they are greater at point A than at points B and C B) they are the same at points A, B, and C C) they are greater at points A and B than at point C D) they are greater at point C than at points A and B

B; The continuity equation dictates that while the velocity of fluid at point C is greater than at points A and B due to the smaller cross-sectional area at point C, the volume displaced per second must be the same. Choice (B) is correct.

*See #8 on Kaplan Test #1 to see the images* Hyperopia, or farsightedness, is caused when an object at infinity is focused behind the retina. In order to correct hyperopia, which of the following lens types would be the most appropriate? A) *image of a concave/diverging lens B) *image of a convex/converging lens C) *image of a flat rectangular lens (neither converging or diverging) D) *image of a concave/diverging lens (with half of it flat)

B; The defect of hyperopia causes light to be focused behind the retina, when ideally it should be focused on the retina. In other words, the light is not converging quickly enough. A converging lens of the right focal length would remedy this problem by "making up" for inadequacies of the lenses of the eye. Since a converging lens is a convex lens, the correct answer to this question is Choice (B). Choices (A) and (D) are both diverging, concave lenses, and choice (C) is neither converging nor diverging.

The radioactive isotope of lead, Pb, decays to the radioactive isotope of bismuth, Bi. The decay process responsible for this decay is: A) alpha decay B) beta-minus decay C) beta-plus decay D) gamma decay

B; The difference between Pb and Bi isotopes is a proton. In beta minus decay, a neutron in the parent atom becomes a proton in the daughter atom. Additionally, the parent emits a β-particle, which is an electron given the symbol e- or β-. The generic negative beta decay reaction is: (A/Z) X →(A/Z + 1) Y + β-, where X is the parent atom, Y is the daughter atom, A is the mass number (number of protons and neutrons) and Z is the atomic number (number of protons). This fits perfectly with the scenario in the question stem. According to the periodic table, Bi, the daughter atom, has one more proton than Pb, the parent atom. Thus, choice (B) is correct.

*See passage for #29 on Kaplan Test 2 (short)* Based on the information provided in the passage, which of the following elements is the LEAST reactive? A) Na B) Zr C) Cl D) Cu

B; The focus here is on periodic trends — if titanium is an inert metal (as stated in Paragraph 1), elements in the same group will behave similarly. Zr is the only element out of all the answer choices that is in the same group as titanium. Hence choice (B) is correct.

What is the value of ln(10^6)? A) 6.0 B) 13.8 C) 464.2 D) 5x10^5

B; The natural log of x, ln(x) is equal to log(x) multiplied by a factor of 2.303, ln(x) = (2.303)log(x). This means that the value of ln(10^6 ) = (2.303)log(10^6 ) = 2.303 x 6 ≈ 7/3 x 6 = 14, this matches choice (B) .

*See passage for #18 on Kaplan Test 2 (short)* In figure 2, isomers II and III have a different cyclic form than isomers IV and V. This is because, when compared to the nucleophile of reactions IV and V, the nucleophile in the hemiketal reaction that creates II and III from the straight-chain isomer: A) is a substituent on a lower-numbered carbon, and it forms a pyranose B) is a substituent on a lower-numbered carbon, and it forms a furanose C) is a substituent on a higher-numbered carbon, and it forms a pyranose D) is a substituent on a higher-numbered carbon, and it forms a furanose

B; The number of the hydroxyl group that attacks the carbonyl carbon in the hemiketal reaction to form the ring structure determines the size of the ring. In isomers II and III, this hydroxyl must have been one carbon closer to the carbonyl carbon than it was in isomers IV and V, making a smaller ring. Also, a furanose is a carbohydrate with a 5-membered ring, eliminating answers (A) and (C). Answer (D) is incorrect because we're looking for a lower numbered carbon, not higher, thus answer (B) is correct.

What is the pH of a solution with a hydrogen ion concentration of 10^-6 M? A) 4 B) 6 C) 7 D) 8

B; The pH is defined as the -log [H+ ], so the -log 10^-6 is 6, (B) .

As glucose, a reducing sugar, is oxidized during Fehling's reaction, how is the equilibrium between hemiacetal D-glucose and linear D-glucose shifted? A) the oxidation reaction does not disrupt the equilibrium of glucose B) there is a shift favoring the replacement of the linear form of glucose C) equilibrium favors the formation of the hemiacetal form of glucose D) equilibrium initially favors linear, but eventually reverts to hemiacetal

B; The ring and linear forms of glucose are in equilibrium initially. However, only the linear form can undergo oxidation. Hence, as the Fehling's reaction progresses, more and more of the linear form of glucose gets consumed, causing the equilibrium reaction between the linear and ring form of glucose to shift towards linear glucose. Choice (B) is the only answer choice that matches this prediction.

*See passage for #25 on Kaplan Test 2 (short)* Based on the general theory of enzyme action, as described in the passage, the overall rate of the reaction (at all substrate and enzyme concentrations) is proportional to the concentration of: A) substrate B) ES complex C) enzyme D) product

B; The slower step of the two described in the passage must be the rate-determining step. Therefore, the overall rate of the reaction is proportional to the concentration of ES formed (because it is the compound that reacts in the second step). Choice (B) is correct.

What is the standard enthalpy change, ΔH°, of the reaction below? 2CO(g) + O2 (g) -> 2CO2 (g) A) ΔHf(CO2) - [ΔHf(CO) + ΔHf(O2) ] B) 2 ΔHf(CO2) - 2ΔHf(CO) C) ΔHf(CO) - ΔHf(CO2) D) 2 ΔHf(CO2) + 2 ΔHf(CO)

B; The standard enthalpy change, ΔH° can be determined by using Hess' Law where the sum of the heat of formation of products minus the sum of the heat of reactants will equal the heat of the reaction. The heat of formation for elements is zero so ΔH° = 2ΔHf (CO 2 ) - 2ΔHf (CO), or (B) , based on the coefficients from the balanced reaction. The heat of formation for oxygen gas is not included in the enthalpy calculations because oxygen gas is in its standard state in the chemical reaction, and the standard enthalpy of formation for an element in its standard state is zero.

In the absence of a pH meter, which of the following indicator solutions is most appropriate for a titration of stomach acid with NaOH? Indicator Transition Range I: Gentian violet 0.0-2.0 II: Phenol red 6.4-8.0 III: Indigo carmine 11.4-13.0 A) I only B) II only C) III only D) I, II, or III

B; The stomach produces HCl, a strong acid. The titrant is NaOH, a strong base. For this titration, the pH of the equivalence point is 7. Locate the region of the table that has 7 within the response range for the indicator. In this case, only one answer choice, phenol red, (choice (B)) contains the appropriate pH. (If there were not such a good match in choice (B), then the indicator with the narrowest range around 7 would be most appropriate.)

The isoelectric focusing points pI for four proteins are shown in the following table. Protein -- pI A -- 8.8 B -- 10.2 C -- 4.3 D -- 5.5 At which buffer pH would two out of four of the proteins adhere to a cation-exchange column? A) 3.0 B) 7.0 C) 9.5 D) 11

B; This is a Biochemistry question that falls under the content category "Separation and purification methods." The answer to this question is B because a cation-exchange column only binds to positively charged proteins, which only occurs when the pH is less than the pI. At pH 7.0, both proteins A and B would be positively charged. With distractors A, C, and D, there would be four, one, and zero positively charged proteins, respectively. It is a Reasoning about the Design and Execution of Research question because it inquires about the experimental conditions needed to achieve a desired result.

*See passage for #22 on AAMC Test 1* Compared to micellular Compound 1, Compound 2 is structurally more rigid as a result of what type of interaction? A) intermolecular hydrogen bonding B) intermolecular covalent bonding C) intramolecular hydrogen bonding D) intramolecular covalent bonding

B; This is a Biochemistry question that falls under the content category "Structure, function, and reactivity of biologically-relevant molecules." The answer to this question is B. It can be reasoned that the interaction described is intermolecular in nature, since multiple molecules of micellular Compound 1 come together to form Compound 2, which is a solid. Multiple pieces of information point to the fact that the interaction is disulfide bond formation, including the fact that an oxidant causes the formation of Compound 2, which can be reversed by the addition of a reducing agent. It is a Scientific Reasoning and Problem Solving question because you are asked to bring together theory and observations to draw a conclusion

*See passage for #23 on AAMC Test 3* The glucose meter measures the current produced during Reaction 2. If 0.67 μmol of electrons were measured, what mass of glucose was present in the sample? (Note: The molar mass of glucose is 180 g/mol = 180 μg/μmol.) A) 20 μg B) 60 μg C) 90 μg D) 270 μg

B; This is a General Chemistry question that falls under the content category "Electrochemistry and electrical circuits and their elements." The answer to this question is B because the stoichiometry of the reaction is 2 mol e- per mole of glucose consumed. The device measured 0.67 (2/3) μmol of electrons, indicating that 0.33 (1/3) μmol of glucose was consumed. This weighs 60 μg, based on its molar mass of 180 g/mol. It is a Scientific Reasoning and Problem Solving question because you must determine and use a scientific formula to solve a problem; (0.67 μmol of electrons/1)(1 μmol of glucose/2 μmol of electrons)(180 μg glucose/1 μmol glucose)

*See passage for #53 on AAMC Test 1* When a strip of Zn is placed in a beaker containing 0.1M HCl, H2 (g) evolves. If a strip of Al is placed in a beaker containing 0.1M HCl, does H2 (g) evolve? A) yes; Al is reduced and H+ (aq) is oxidized B) yes; Al is oxidized and H+ (aq) reduced C) no; Al is reduced and Cl- (aq) is oxidized D) no; Al is oxidized and H2O (l) is produced

B; This is a General Chemistry question that falls under the content category "Electrochemistry and electrical circuits and their elements." The answer to this question is B. Since a new solid forms when Al(s) is mixed with Zn2+(aq), it is reasonable to assume that Al(s) is more susceptible to oxidation than Zn(s). It is a Scientific Reasoning and Problem Solving question because you are asked to bring together theory and observations to draw a conclusion.;Al(s) + Zn2+(aq) forms a solid, meaning that Zn2+ is reduced. Zn(s) + Al3+(aq) does not, meaning that Al3+ is not reduced. The conclusions we can draw from this are 1) that Zn2+ has a higher reduction potential than Al3+ and 2) that Al(s) has a higher oxidation potential than Zn(s). These are very similar to the conclusions you described, but note that I've made sure to compare two like things!So since H2 gas evolves when in the presence of Zn(s), and Al(s) has an even higher oxidation potential than Zn(s), we know that H2 gas must evolve - or in other words, H+ must be reduced - when in the presence of Al(s).

*See passage for #19 on AAMC Test 1* What percentage of standard atmospheric pressure is the pulse pressure of a healthy adult? A) 10% B) 6% C) 2% D) 1%

B; This is a General Chemistry question that falls under the content category "Importance of fluids for the circulation of blood, gas movement, and gas exchange." The answer to this question is B because the pulse pressure in a healthy adult is (120 − 75) mmHg = 45 mmHg, and so the percentage is 45 mmHg/760 mmHg = 6%. It is a Knowledge of Scientific Concepts and Principles question because you are asked to use mathematical equations to solve problems.

How will the rate of a catalyzed reaction be affected if the solid catalyst is finely ground before it is added to the reaction mixture? A) the rate will be faster because a greater mass of catalyst will be present B) the rate will be faster because a greater surface area of catalyst will be exposed C) the rate will be slower because the fine catalyst particles will interfere with product formation D) the rate will remain the same because the mass of catalyst will be the same

B; This is a General Chemistry question that falls under the content category "Principles of chemical thermodynamics and kinetics." The answer to this question is B because grinding a heterogeneous catalyst increases the amount of catalyst available to the reaction and therefore increases its rate. It is a Reasoning about the Design and Execution of Research question because you are asked to identify the relationship among variables in a study (surface area of catalyst and rate).

*See passage for # 37 on AAMC Test 2* Consider the reaction shown in Equation 1 at equilibrium. Would the concentration of [Cu(NH3)4]2+ increase if the equilibrium were disturbed by adding hydrochloric acid? A) yes, because the equilibrium in Equation 1 would shift to the left B) no, because the equilibrium in Equation 1 would shift to the left C) yes, because the equilibrium in Equation 1 would shift to the right D) no, because the equilibrium in Equation 1 would shift to the right

B; This is a General Chemistry question that falls under the content category "Principles of chemical thermodynamics and kinetics." The answer to this question is B. Hydrochloric acid will protonate ammonia in a Brönsted acid-base reaction and reduce the amount of ammonia present. The disturbed equilibrium responds in a way to restore ammonia, but this causes the amount of [Cu(H2O)2(NH3)2]2+ to decrease. This means that the equilibrium shifts to the left. It is a Scientific Reasoning and Problem Solving question because you must use Le Châtelier's principle as well as the behavior of acids and bases to draw a conclusion about the effect of adding hydrochloric acid to a reaction.

*See passage for #50 on AAMC Test 1* Which of the following most likely will occur if a homogeneous catalyst CANNOT be separated from the products at the end of a reaction? A) the catalyst will become heterogeneous B) the products will be contaminated C) the reaction will not occur D) the reaction rate will speed up

B; This is a General Chemistry question that falls under the content category "Principles of chemical thermodynamics and kinetics." The answer to this question is B. If a homogeneous catalyst cannot be separated from the products at the end of a reaction then the products will be contaminated with the catalyst. It is a Reasoning about the Design and Execution of Research question because you are asked to reason about the features of a particular set of reaction conditions and their effects on the outcome (product purity).

In [Cu(NH3)4]^2+, the subscript 4 indicates which of the following? A) The oxidation number of Cu only B) The coordination number of Cu^2+ only C) Both the oxidation number of Cu and the coordination number of Cu^2+ D) Neither the oxidation number of Cu nor the coordination number of Cu^2+

B; This is a General Chemistry question that falls under the content category "Unique nature of water and its solutions." The answer to this question is B. Because ammonia is neutral, the number 4 reflects only the number of ammonia molecules that bind to the central Cu^2+ cation and does not indicate anything about its oxidation number. It is a Knowledge of Scientific Concepts and Principles question because you must recognize a correct scientific principle.

Each of the following equations shows the dissociation of an acid in water. Which of the reactions occurs to the LEAST extent? A) HCl + H2O -> H3O+ + Cl- B) HPO4 ^2- + H2O -> H3O+ + PO4 ^3- C) H2SO4 + H2O -> H3O+ + HSO4 - D) H3PO4 + H2O -> H3O+ + H2PO4 -

B; This is a General Chemistry question that falls under the content category "Unique nature of water and its solutions." The answer to this question is B. HPO42− has a high negative charge and so dissociation of it will occur to the least extent. It is a Scientific Reasoning and Problem Solving question because you are asked to use a scientific principle and theory to draw a conclusion.

What is the pH of a buffer solution that is 0.2 M in HCO3- and 2 M in H2CO3? (Note: The first pKa of carbonic acid is 6.37.) A) 4.37 B) 5.37 C) 6.37 D) 7.37

B; This is a General Chemistry question that falls under the content category "Unique nature of water and its solutions." The answer to this question is B. The pH of the solution can be calculated using the Henderson-Hasselbach equation: pH = pKa + log([base]/[acid]). Plugging in the values provided in the question gives pH = 6.37 + log(0.2/2) = 5.37. It is a Scientific Reasoning and Problem Solving question because you determine and use scientific formulas to solve a problem.

*See passage for #15 on AAMC Test 2* Based on the relative energy of the absorbed electromagnetic radiation, which absorber, a peptide bond or an aromatic side chain, exhibits an electronic excited state that is closer in energy to the ground state? A) an aromatic side chain; the absorbed photon energy is higher B) an aromatic side chain; the absorbed photon energy is lower C) a peptide bond; the absorbed photon energy is higher D) a peptide bond; the absorbed photon energy is lower

B; This is a Physics question that falls under the content category "Atoms, nuclear decay, electronic structure, and atomic chemical behavior." The answer to this question is B because aromatic side chains absorb in the near UV region of the electromagnetic spectrum, which has longer wavelengths, and hence lower energy, than peptide bonds. Because the energy of the photon matches the energy gap between the ground and the excited state, this implies that the aromatic side chain has more closely spaced energy levels. It is a Knowledge of Scientific Concepts and Principles question because the question requires identifying the relationships among closely related concepts.

*See passage for #5 on AAMC Test 3* What is the total amount of charge and energy, respectively, that the capacitor will store if it is connected to the battery on the railcar? A) 0.12 C and 14.4 J B) 1.20 C and 7.20 J C) 1.44 C and 0.12 J D) 7.20 C and 1.20 J

B; This is a Physics question that falls under the content category "Electrochemistry and electrical circuits and their elements." The answer to this question is B because the charge is given by the relation 100 mF × 12 V = 1.2 C and the energy is equal to 0.5 × 100 mF × (12 V)^2 = 7.2 J. It is a Scientific Reasoning and Problem Solving question because it asks you to determine and use scientific formulas to solve problems.

Knowing that the speed of light in the vitreous humor is 2.1x10^8 m/s, what is the index of refraction of the vitreous humor? (Note: The speed of light in a vacuum is 3.0x10^8 m/s.) A) 0.7 B) 1.4 C) 2.1 D) 3.0

B; This is a Physics question that falls under the content category "How light and sound interact with matter." The answer to this question is B because the index of refraction of a medium is equal to the ratio of the speed of light in vacuum to the speed of light in the medium, thus it is equal to (3.0 x 108 m/s)/(2.1 x 108m/s) = 1.4. It is a Scientific Reasoning and Problem Solving question because you are asked to determine and use scientific formulas to solve problems.

*See figure for #45 on AAMC Test 1* In the above figure, an object O is at a distance of three focal lengths from the center of a convex lens. What is the ratio of the height of the image to the height of the object? A) 1/3 B) 1/2 C) 2/3 D) 3/2

B; This is a Physics question that falls under the content category "How light and sound interact with matter." The answer to this question is B because the ratio of the image height to the object height is equal to the ratio of the lens-image distance to the object-lens distance. According to the thin lens equation, the distance between the lens and the image is 3F x 1F/(3F - 1F) = (3/2)F, where F is the focal length. The ratio sought is then equal to (3/2)F/(3F) = 1/2. It is a Scientific Reasoning and Problem Solving question because you are asked to determine and use scientific formulas to solve problems.; So you know Object distance = 3f, focal length = f, image distance = i. If you plug this into the formula 1/o + 1/i = 1/f you get 1/3f + 1/i = 1/f. Rearrange you get 1/i = 1/f - 1/3f ==> 1/i = 2/3f ==> i = 1.5f. Plug this into the magnification formula m = -i/o and you get 1.5f/3f which is 1/2!

Suppose that a blood vessel of cross-sectional area A carries microbubbles at a speed v into a capillary bed. If the capillary bed is made up of n capillaries, each with cross-sectional area a, with what speed will the blood flow in the capillary bed? A) (na/A)v B) (A/na)v C) (a/A)v D) (nA/a)v

B; This is a Physics question that falls under the content category "Importance of fluids for the circulation of blood, gas movement, and gas exchange." The answer to this question is B because, according to the continuity equation, if u is the speed of flow in the capillary bed, then A×v = n×a×u, so u = A×v/(n×a). It is a Scientific Reasoning and Problem-solving question because it asks you to determine and use scientific formulas to solve problems.

*See passage for #7 on AAMC Test 3* If no braking occurs, a total of how much power would be required to keep the railcar moving at 40 m/s? A) 16 kW B) 40 kW C) 600 kW D) 800 kW

B; This is a Physics question that falls under the content category "Translational motion, forces, work, energy, and equilibrium in living systems." The answer to this question is B because the power required must match the work done by the friction force that tends to slow down the railcar, which is equal to the decelerating force multiplied by the constant speed, so 1000 N × 40 m/s = 40 kW. It is a Scientific Reasoning and Problem Solving question because it asks you to determine and use scientific formulas to solve problems.

*See passage for #14 on Kaplan Test 2 (short)* Which of the following is/are true about Reaction 1? I. The reaction is not spontaneous II. There is an overall decrease in entropy during the reaction III. The reaction is exothermic A) I only B) I and II only C) II and III only D) I, II, and III

B; This is a Roman numeral item, so first examine statement I because it shows up most frequently in the answer choices. Paragraph 3 says that delta G= +686 kcal/mol. Since the sign is positive, the reaction is not spontaneous. Therefore, statement I is true and choice (C) can be eliminated. Next consider statement II. The change in entropy during the reaction can be estimated by considering two factors: the number of moles of reactants and products and any phase changes. In this case, there are twelve moles of reactants and only seven moles of products. Therefore, there must be a decrease in entropy. Furthermore, there is a change in phase from gaseous reactants to a solid product; this would also correspond to a decrease in entropy. Therefore, statement II is also true and choice (A) can be eliminated. Finally, to examine statement III. Since the reaction is taking in energy in the form of light, it is an endothermic reaction. Therefore, statement III is false, and choice (B) is the correct answer.

*See passage for #30 on AAMC Test 1* The production of a variety of opsins functions to: A) increase sensitivity to low light B) enable the detection of different colors C) ensure fast recovery of 11-cis-retinal after exposure D) increase refractive index of the eye lens

B; This is an Organic Chemistry question that falls under the content category "How light and sound interact with matter." The answer to this question is B. The wavelength of light absorbed by a molecule depends on its structure, and so the production of a variety of structurally related opsins functions to enable the detection of different colors. It is a Scientific Reasoning and Problem Solving question because you are asked to reason using scientific principles.

Which property of a substance is best used to estimate its relative vapor pressure? A) melting point B) boiling point C) molecular weight D) dipole moment

B; This is an Organic Chemistry question that falls under the content category "Separation and purification methods." The answer to this question is B because of the properties listed, the boiling point of a substance will give the best estimate of its relative vapor pressure. It is a Scientific Reasoning and Problem Solving question because you must reason using the kinetic molecular theory in order to conclude that boiling point and vapor pressure are closely related properties.

When two amino acids are joined via a peptide bond, what is the mass of the byproduct of this reaction? (Note: Assume that the amino acids were not modified by protecting groups.) A) 17 amu B) 18 amu C) 32 amu D) 44 amu

B; This is an Organic Chemistry question that falls under the content category "Structure, function, and reactivity of biologically-relevant molecules." The answer to this question is B because the formation of a peptide bond is accompanied by the formation of water as a by-product, and the mass of water is 18 amu. It is a Scientific Reasoning and Problem Solving question because one must understand (1) that water is released as a by-product of peptide bond formation from the OH of the carboxyl group and the NH of the amino group, and (2) that the mass of H2O is the sum of its atomic masses.

Which of the following best explains the use of D-amino acids by bacteria in peptidoglycan synthesis? A) Bacterial environments contain only D-amino acids B) D-amino acids are poorly recognized by other species C) D-amino acids have preferable physical properties D) D-amino acids can be easily substituted with L-amino acids

B; This question incorporates some basic biological knowledge with your background in stereochemistry. We are looking for an answer that is advantageous to the bacterium by preventing its degradation or enhancing pathogenicity. This is a vague prediction, but matches choice (B). Choice (A) cannot be true because bacteria can live a wide variety of environments and will have a variety of amino acids available to them. Choice (C) is wrong because enantiomers have the same physical properties with the exception of the direction of polarization for plane polarized light which would confer no plausible advantage. Choice (D) is wrong because enzymes are typically stereospecific, and hence this statement is unlikely to be true.

*See image on #30 on Kaplan Test #1* Which of the following statements correctly describes the separation of the compounds below by extraction into dichloromethane and water? A) if the solution were acidic, caffeine would be more soluble in dichloromethane than in water B) if the solution were basic, caffeine would be more soluble in dichloromethane than in water C) if the solution were basic, octanoic acid would be more soluble in dichloromethane than in water D) if the solution were acidic, octanoic acid would be more soluble in water than in dichloromethane

B; Under basic conditions, the caffeine molecule is deprotonated and so is neutral. As a result, it will be more soluble in organic solvents (like dichloromethane) rather than in water. Under the same conditions, octanoic acid is in its charge or polar form and would therefore be more soluble in water than in dichloromethane. Under basic conditions, then, caffeine would be found in the organic layer, not the aqueous layer, which means that choice (B) is the correct answer. Under acidic conditions, caffeine is protonated (on the nitrogen and possesses a positive charge, while octanoic acid is protonated and is neutral. Therefore, caffeine would be more soluble in water, nit in dichloromethane (so choice (A) is incorrect). As for octanoic acid, in a basic solution, octanoic acid will be deprotonated and will be negatively charged. As a result, it will be more soluble in the aqueous phase than the organic phase, and so choice (C) is incorrect. Conversely, in an acidic solution, octanoic acid will be protonated and neutral. It will therefore be more soluble in dichloromethane than water, and so choice (D) is incorrect.

A horizontal steel bar of uniform mass density and mass 5kg is supported by two vertical steel cables attached to the ends of the bar. When a 75kg mass hangs from the center of the bar, the tension in each cable is: A) 75 N B) 375 N C) 400 N D) 800 N

C; A free-body diagram would be helpful for this question. The two downward forces are the steel bar weighing 5 kg and the 75 kg mass, to total approximately 80 kg or a 800 N downward force. Since the steel bar is stationary, the tension in the cables must equal the downward 800 N force. Since there are two cables, the tension gets divided by each. Therefore, the tension in each cable is 400 N each —choice (C) .

*Look at #23 on Kaplan Test #1* During an execution of Method 2 from the passage assume that a mass m of 0.2 kg is placed 25 cm to the right of the fulcrum. A section of gut initially weighing 0.1 kg is placed 50 cm to the left of the fulcrum. During the experiment, the mass is seen to descend. in order to maintain the balance level the following action should be taken: A) the mass m should be moved away from the fulcrum B) the arterial pressure should be decrease C) the venous pressure should be increased D) an inert fluid with high density should be used in the pipette

C; According to the passage when fluid flows out of the gut (because the hydrostatic pressure in the capillary bed is lower than the osmotic pressure), the gut will become lighter and will ascend. Thus, the mass m will descend. The question stem indicates that the section of gut has become lighter. Fluid must be returned to the gut section to make it heavier and maintain the balance level. Increasing the venous pressure, choice (C), will increase the hydrostatic pressure in the capillary bed, leading to the flow of fluid from the capillary into the gut section. Note that no quantitative analysis (calculation) is necessary to answer this question. As for the incorrect answer choices, (A) is incorrect because moving the mass away from the fulcrum will increase the distance between the force applied and the fulcrum, thus increasing the torque produced by the mass m causing it to descend more. (B) is incorrect because decreasing the arterial pressure will decrease the hydrostatic pressure in the capillary bed causing more fluid to be lost from the gut. The gut will become lighter and will ascend, causing the mass to descend more. Finally, (D) is incorrect because the pipette (from Method 1) does not come into play in this set-up, which deals with Method 2.

*See passage for #37 on Kaplan Test #1* Which of the following cellular mechanisms would be directly affected by a vitamin B12 deficiency? A) DNA replication only B) RNA synthesis only C) Both DNA replication and RNA synthesis D) Neither DNA replication nor RNA synthesis

C; Both DNA and RNA synthesis require purines, thus both would be affected by a vitamin B12 deficiency and choice (C) is correct.

Which of the following carboxylic acid derivatives are most reactive towards nucleophilic acyl substitution? A) esters B) acid anhydrides C) acyl halides D) amides

C; Carboxylic acid derivatives are acyl groups attached to halides (acyl halides), carboxyl group (acid anhydrides), amine (amides), or alkoxy groups (esters). The reactivity of a carboxylic acid derivative in nucleophilic acyl substitution is governed by the strength of the leaving group. Acyl halides are the most reactive carboxylic acid derivative and therefore the least stable, while amides are the least reactive and the most stable. Acid anhydrides are more reactive than esters, but less reactive than acyl halides. Therefore, the order of reactivity is acyl halides > acid anhydrides > esters > amides. Choice (A) . Esters are one of the more stable carboxylic acid derivatives and therefore not as reactive as acyl halides or acid anhydrides. The leaving group for an ester is an alkoxy group (-OR), which is a poor leaving group. Choice (B) . Acid anyhydrides are the second most reactive carboxylic acid derivative, but not as strong as acyl halides. Choice (D) . Amides are the least reactive and the most stable carboxylic acid derivative.

How would the 1H NMR change if ethane, CH3CH3, were changed to chloroethane, CH3CH2Cl? A) one singlet would become two doublets with one doublet shifted downfield B) one singlet would become a doublet and a triplet with the triplet shifted downfield C) one singlet would become a triplet and a quartet, with the quartet shifted downfield D) two quartets would become a triplet and a quartet, with the quartet shifted downfield

C; In 1H NMR spectroscopy, it is important to look at the different kinds of hydrogen present. In ethane, all of the hydrogen are identical which means that there will only be one singlet, a large peak, and since it is an alkane, it will be not very far downfield. In chloroethane, there are now two different types of hydrogen. The hydrogen closest to the chlorine should be deshielded by the electronegative character of chlorine and therefore be shifted downfield. The next thing to look at is splitting. Splitting follows the n+1 rule, where a type of hydrogen will be split by how many hydrogens are on adjacent carbons. In chloroethane, the hydrogen proximal to the chlorine have three hydrogen on the adjacent carbon and thus will form a quartet. For the hydrogen distal to chlorine, they have two hydrogen on an adjacent carbon and will therefor split into a triplet. These two factors match choice (C) .

An object is placed in between the focal point and the surface of a thin converging lens. The image produced is: A) real and upright B) real and inverted C) virtual and upright D) virtual and inverted

C; Information about the image (i) can be found using the equation 1/i = 1/f - 1/o. Since the lens is converging, the focal length takes a positive sign. The object (like all objects in a single lens system) has a positive sign. Since we are told f > o, the value of 1/i, and thus i, is negative. The negative sign of i indicates that the image is virtual. To find out whether the image is upright or inverted, calculate the magnification, m = - i/o. Since i is negative and o is positive, the magnification is positive. Thus, the image is upright. This brings gives choice (C) .

Ozone has a high oxidation potential, and will readily decompose urea: (NH2)2CO + O3 -> N2 + CO2 +2H2O. In this reaction, what is the oxidation number of carbon as a reactant and product, respectively? A) -2 and +4 B) +2 and +4 C) +4 and +4 D) +4 and -4

C; It is simpler to figure out the oxidation number of carbon in CO2, so begin there. Oxygen has an oxidation number of -2, and there are two oxygens, for a total of -4. To balance this, the oxidation number of carbon must be +4, and so choice (D) can be eliminated. Next, determine the oxidation number of carbon in urea, by finding the oxidation numbers for all the other molecules. Nitrogen has an oxidation number of -3, and there are two nitrogens, for a total of -6. Hydrogen has an oxidation number of +1, and there are 4 hydrogens, for a total of +4. Finally, oxygen has an oxidation number of -2. The total oxidation numbers for nitrogen, hydrogen, and oxygen can be added up: -6 + 4 -2= -4. Since urea is neutral, C will be +4. Match to choice (C).

Which of the following cannot be oxidized to form a carboxylic acid? A) a primary alcohol B) an alkyl benzene C) a ketone D) an aldehyde

C; One of the common ways to synthesize carboxylic acids is to use a strong oxidizing agent, such as PCC or potassium permanganate (KMnO 4 ). Secondary or tertiary alcohols and ketones would be unsuitable as a starting reagent to produce a carboxylic acid because of valence limitations. Choice (A) . A primary alcohol can be oxidized into a carboxylic acid since it terminates in a -OH group. Choice (B) . An alkyl benzene can be converted to a carboxylic acid by oxidation. Choice (D) . An aldehyde can be converted into a carboxylic acid by oxidation.

A scientist wishes to test the effects of various solutes on osmotic pressure. She has two chambers filled with distilled water and separated by a semipermeable membrane. In each trial, she adds 0.2 moles to one of the chambers and measures the osmotic pressure. Which of the following solutes will produce the greatest change in osmotic pressure? A) NaCl B) HCO3 - C) CaCl2 D) Glucose

C; Osmotic pressure is a colligative property, meaning it is a property that is dependent upon the number of particles present and, ideally speaking, not the identity of those particles. The answer that would affect osmotic pressure the most is the compound that would produce the greatest number of particles. Equal amounts (.2 moles) of each is added, so the number of particles will depend upon the solubility and dissociation products. Choice (C)will produce the greatest number of particles since it is soluble and dissociates into three particles. Choice (A)While soluble, sodium chloride will only dissociate into two particles. Choice (B)The bicarb ion is a weak acid (and weak base) and will only dissociate slightly. Choice (D)Glucose is a covalently bound molecule that will not dissociate. Choice (C) is the correct answer.

Which of the following is NOT equivalent to log (XY/WZ)? A) log(X) + log (Y) - log(W) - log(Z) B) log(X) + log (Y) + log(1/W) - log(Z) C) log(Y) - (log(WZ)+ log(X)) D) log(X) - (log(W) + log (Z) - log(Y))

C; Properties of logs state that log(1/x) = -log(x) and log(xy) = log(x) + log(y). Using these properties shows that the only non-equivalent statement is choice (C) since it is equivalent to log(Y/WZX).

A scientist wants to prepare a 50 mM solution of NaOH. If he starts out with 60 mL of 0.3 M NaOH and adds pure water until reaching the desired concentration, how much water will he have added? A) 30 mL B) 36 mL C) 300 mL D) 360 mL

C; Since only pure water is being added, the number of moles of NaOH should stay the same which means Molarity x Volume = Constant. This means M 1 V 1 = M 2 V 2 or (60 mL)(0.3 M) = (0.05M)(V 2 ), solving for V 2 gives V 2 = 360 mL. Since the scientist started with 60 mL that means he had to add 300 mL of water to dilute the solution, or choice (C)

Which aqueous reagent would be most suitable in the extraction of phenol from CCl4? A) sodium bicarbonate B) hydrochloric acid C) sodium hydroxide D) sodium chloride

C; The basic scheme behind an organic extraction is to isolate the organic molecule from the organic layer by moving it into the aqueous layer by protonation with an acid or deprotanation with a base. The aqueous layer can then be easily drained from the seperatory funnel and the organic molecule can be isolated. Phenol is a weakly acidic organic molecule, so only a strong base will deprotonate the molecule. If a weak base is used, it will not be sufficient to extract the proton from phenol, so it will remain in the organic CCl 4 layer. Of the choices listed, only sodium hydroxide is a strong base. Choice (A) . Sodium bicarbonate is a weak base and therefore will not deprotonate phenol so that it goes to the aqueous layer. Weak bases are used to deprotonate strong organic acids in extractions. Choice (B) . Phenol is a weak acid so a strong acid like HCl would be ineffective in an extraction. Choice (D) . Sodium chloride is neither an acid nor a base, so it would have no effect in an extraction.

A compound with the general formula RCOOCOR (where R= -H or any alkyl group) is classed as a(n): A) ether B) ester C) acid anhydride D) epoxide

C; The compound has two carbonyl oxygens with a shared oxygen between the two carbonyl carbons. That is the general form of an acid anhydride that results from the reaction of two carboxylic acids with the loss of water between them. Choice (A) . An ether has no carbonyl groups, so this cannot be correct. The general form of an ether is (ROR'). Choice (B) . An ester has only one carbonyl group and results from the reaction of a carboxylic acid and an alcohol. Choice (D) . An epoxide is a cyclic ether. They do not contain carbonyl groups.

*See #45 on Kaplan Test 2 (short)* Which of the following best represents the enolate ion intermediate of the racemization shown in Figure 3? A) *see image B) *see image C) *see image D) *see image

C; The enolate anion is stabilized by resonance, which delocalizes the negative charge to the electronegative oxygen. Therefore, choice (C) is the correct answer. Choice (A) is incorrect because it shows delocalization of a positive charge. Choice (B) is incorrect because although this enolate anion may form through abstraction of an alpha proton from the methyl group, it will not lead to racemization of the chiral center. Note that the structure in Choice B maintains a chiral center which has not been turned into an achiral, planar configuration which would lead to racemization. Finally, choice (D) is incorrect because the methyl group would not be deprotonated.

A 10 kg object slides down an incline that has a coefficient of friction of 0.3. As the angle of the incline increases, the frictional force: A) stays the same B) increase C) decreases D) increases for inclinations below 45 degrees and then decreases for inclinations above 45 degrees

C; The frictional force, Fk = μk F N , is dependent on the normal force, F N . When the object is placed on a horizontal surface, F N cos(θ) is equal to F N . As θ increases, the normal force approaches 0 as the plane surface becomes vertical, so as the angle increases the frictional force decreases or (C) .

The human head weighs 4.8kg and its center of mass is 1.8cm in front of the spinal column joint. If the trapezius muscle inserts 1.1cm behind the spinal cord joint, how much downward force does it need to exert in order the maintain equilibrium? A) 7.7 N B) 11 N C) 77 N D 110 N

C; The human skull resting on top of the spinal column can be modeled as a lever in rotational equilibrium. In this model of a lever, the joint between the skill and spinal column is acting as the fulcrum, the weight of the head is producing a downward force on one side of the fulcrum, and the trapezius muscle is producing a downward force on the other side of the fulcrum. The torque due to the weight of the head is (4.8kg)(9.8 m/s^2)(0.018m). In order to maintain rotational equilibrium this torque needs to be balanced by (F)(0.011m) where F is the force applied by the trapezius muscle. Solving for F gives 77 N, which matches with choice (C). Note that this can also be estimated fairly accurately by making the following approximations: 4.8kg= ~5kg 9.8m/s= ~10m/s 1.1cm= ~1.2cm (which is exactly 2/3rds of 1.8cm) This would make the answer require little to no scratch-paper calculation. The particularly temping answer choices are choice (A), reached by calculating with the mass of the head instead of the force due to the mass of the head (fail to multiply mass by 9.8 m/s^2), and choice (D) which can be reached by rounding too carelessly.

An adult patient is considering LASIK surgery, but is concerned about the likelihood of a successful procedure given that she only recently developed myopia (the condition the surgery corrects). Which of the following would provide the best data to address this patient's concerns? A) A survey of those with myopia differentiating the percentage that developed myopia in childhood, adolescence, and adulthood. B) An insurance company's report noting how many times her surgeon had been sued for malpractice related to LASIK surgeries C) A longitudinal study of those experiencing similar pathogenesis and etiology that details treatment elections and outcomes. D) A review of successful LASIK surgeries, noting the percentage of patients who were female and had developed myopia in adulthood

C; The longitudinal study, choice (C), will best provide information on the effectiveness and possible side effects from the surgery. Because it includes patients from across the country, presumably it will find some in her condition (female, adult onset myopia). Each of the other choices falls short of what the patient wants. Choice (A) doesn't observe outcomes of LASIK. Choice (B) may provide some insight into the skill of her surgeon, but will not provide insight into her predicted success or non-effective surgery that didn't result in a malpractice suit. Choice (D) is a retroactive study that only considered patients with successful outcomes, so there is no information on similar patients whose surgery didn't work.

Which of the following factors can change the numerical value of an equilibrium constant Keq? A) volume B) concentration C) temperature D) pressure

C; The only physical factor that can change the value of the Keq is temperature. None of the other factors listed will affect the value for Keq . Since the Ka , Kb , Ksp , are all Keq , only temperature will change the values of these constants. This matches choice (C) .

*See #26 on Kaplan Test #1* Assume that the beam balance of Method 2 is initially level. If the arterial pressure is decreased to a lower level while everything else is held constant, which graph best represents the change in the mass of the gut following the decrease in arterial pressure? A) mass of gut decreases as time increases (linear) B) mass of gut increases as time increases (linear) C) mass of gut decreases for a little and then levels off as time increases D) mass of gut is leveled off as time increases, until it reaches a certain time at which the mass drops off

C; The passage states that if the arterial pressure is decreased, fluid will leave the gut section and its mass will decrease. The mass will decrease until a certain amount of fluid is lost and a new equilibrium between hydrostatic pressure and osmotic pressure in the capillaries is reached. Choice (C) best shows the initially rapid decrease in mass and the gradual leveling out as a new equilibrium is reached. Choice (A) is incorrect because the mass of the gut will not decrease to zero. Also, the rate of mass loss will not be linear. There will be a rapid mass loss followed by a gradual leveling out. Choice (B) is incorrect because the mass of the gut will decrease as fluid is lost from the gut section to the capillaries. Choice (D) is incorrect because the mass loss will occur rapidly at first, followed by a gradual leveling out as a new equilibrium is reached. Choice (D) indicates a gradual increase in the rate of mass loss, which is the opposite of what actually happens.

0.44 moles of AgNO3 and 0.20 moles of MgCl2 are mixed together. What is the mass percent composition of Ag in the product? A) 0.8% B) 25.7% C) 75.2% D) 90.9%

C; The starting number of moles is irrelevant information when calculating the mass percent composition. The products of the reaction are AgCL and Mg(NO3)2. Since the question stem asks about the mass percent composition of Ag, the product of interest is AgCl. It doest not matter how many moles of AgCl are formed; there will always be 107.9g of Ag per mole of product and 35.5g of Cl per mole of product, since those are the molar masses of Ag and Cl respectively. The total mass of AgCl is 143.3g/mol, so the mass percent composition of Ag is 107.9/143.3 or 75.2%. This matches choice (C).

*See passage for #41 on AAMC Test 1* Addition of laforin to the reaction mixtures used to generate the data in Figure 2 will result in: A) a decrease in the intensity of the band representing WT without glucosidase when visualized with 14C decay B) an increase in the intensity of the band representing WT without glucosidase when visualized with 14C decay C) a decrease in the intensity of the band representing WT without glucosidase when visualized with 32P decay D) an increase in the intensity of the band representing WT without glucosidase when visualized with 32P decay

C; This Biochemistry question falls under the content category "Principles of chemical thermodynamics and kinetics." The answer is C because laforin is an enzyme that removes phosphate from glycogen. Presumably, the removal of phosphate prior to gel electrophoresis will result in a smaller band corresponding to 32P incorporation. This is a Reasoning about the Design and Execution of Research question because you are asked to make a prediction based on the properties of the experimental design.

*See passage for #34 on AAMC Test 1* Which reaction is catalyzed by LipA? A) ATP hydrolysis B) Peptide bond cleavage C) Hydrolysis of triacylglycerides D) Transfer of carboxyl groups

C; This is a Biochemistry question that falls under the content category "Principles of chemical thermodynamics and kinetics." The answer is C because of the fact that LipA is a lipase, which means it must hydrolyze fatty acids. This is a Knowledge of Scientific Concepts and Principles question because you are asked to identify the role of a lipase in biochemical reactions.

*See passage for #32 on AAMC Test 1* The source of the phosphate groups that are added to rhodopsin is: A) arrestin B) rhodopsin kinase C) ATP D) all-trans-retinol

C; This is a Biochemistry question that falls under the content category "Principles of chemical thermodynamics and kinetics." The answer is C because the requirement of ATP for kinase activity implies the phosphate groups come from ATP. All the other responses do not have phosphate to transfer. It is a Knowledge of Scientific Concepts and Principles question because you are asked to recall and apply the concept of phosphate group transfer to a particular enzyme substrate relationship.

*See passage for #18 on AAMC Test 2* In designing the experiment, the researchers used which type of 32P labeled ATP? A) alpha 32P-ATP B) beta 32P-ATP C) gamma 32P-ATP D) delta 32P-ATP

C; This is a Biochemistry question that falls under the content category "Principles of chemical thermodynamics and kinetics." The answer to this question is C because the phosphoryl transfer from kinases comes from the gamma-phosphate of ATP. Therefore, the experiment should require gamma 32P-ATP. It is a Reasoning about the Design and Execution of Research question because you must determine the correct way of conducting an experiment to test the desired hypothesis.

*See passage for #14 on AAMC Test 2* Which amino acid will contribute to the CD signal in the far UV region, but NOT the near UV region, when part of a fully folded protein? A) Trp B) Phe C) Ala D) Tyr

C; This is a Biochemistry question that falls under the content category "Structure, function, and reactivity of biologically-relevant molecules." The answer to this question is C because all chiral nonaromatic amino acids will contribute solely to the CD signal in the far UV region. It is a Knowledge of Scientific Concepts and Principles question because you must identify the relationship between closely related concepts.

*See passage for #43 on AAMC Test 3* Based on the information in the passage and in Figure 1, what effect does epigenetic modification have on iM pH-dependent denaturation? A) Both methylation and hydroxymethylation result in significantly decreased stability because cytosine is more readily deprotonated. B) Both methylation and hydroxymethylation result in significantly decreased stability because cytosine is more readily protonated. C) Only hydroxymethylation results in significantly decreased stability because cytosine is more readily deprotonated. D) Only hydroxymethylation results in significantly decreased stability because cytosine is more readily protonated.

C; This is a Biochemistry question that falls under the content category "Structure, function, and reactivity of biologically-relevant molecules." The answer to this question is C because the drop in pK of the transition denotes a decrease in stability. Because DNA unfolding occurs as the pH increases, it can be inferred that this is due to cytosine deprotonation. Since 5hmC-WT has the lowest pK, hydroxymethylation decreases the stability by increasing the acidity of cytosine. It is a Data-based and Statistical Reasoning question because it requires interpreting graphical data in order to draw scientific conclusions.

*See passage for #25 on AAMC Test 1* Compound 1 was designed to exhibit pH-dependent self-assembly. What feature(s) of the molecule is(are) responsible for the pH dependence of aggregation (Equation 1)? A) Thiol side chains that can hydrogen bond B) long alkyl tail that exhibits predominantly London forces C) side chains whose net charge responds to pH D) covalent linkages that reversibly hydrolyze

C; This is a Biochemistry question that falls under the content category "Structure, function, and reactivity of biologically-relevant molecules." The answer to this question is C because the molecule contains mainly acidic side chains that were deprotonated and negatively charged at high pH. This inhibited aggregation due to electrostatic principles. At low pH these groups are neutral and this allowed aggregation. It is a Reasoning about the Design and Execution of Research question because you are asked to identify the relationship among variables in a study (pH and solubility).

*See passage for #54 on AAMC Test 1* Assume that a circuit similar to that in Figure 2 is set up in which X=Al, X^n+ =Al^3+, Y=Cu, and Y^m+ = Cu^2+. Which of the following reactions will occur? A) 2Al^3+ (aq) + 3Cu (s) -> 2Al (s) + 3Cu^2+ (aq) B) 3Al^3+ (aq) + 2Cu (s) -> 3Al (s) + 2Cu^2+ (aq) C) 2Al (s) + 3Cu^2+ (aq) -> 2Al^3+ (aq) + 3Cu (s) D) 3Al (s) + 2Cu^2+ (aq) -> 3Al^3+ (aq) + 2Cu (s)

C; This is a General Chemistry question that falls under the content category "Atoms, nuclear decay, electronic structure, and atomic chemical behavior." The answer to this question is C because this response depicts the balanced chemical reaction that occurs when the four substances [Al(s), Al3+(aq), Cu(s), and Cu2+(aq)] are mixed. It is a Scientific Reasoning and Problem Solving question because you are asked to bring together theory and observations to draw a conclusion.

*See passage for #16 on AAMC Test 3* What type of reaction is occurring between I2and Zn? A) Acid-base (neutralization) B) Precipitate formation C) Oxidation-reduction D) Chelate formation

C; This is a General Chemistry question that falls under the content category "Atoms, nuclear decay, electronic structure, and atomic chemical behavior." The answer to this question is C, because both zinc and iodine change oxidation states during the reaction. It is a Knowledge of Scientific Concepts and Principles question because you must recognize and identify a reaction as oxidation-reduction based on the concept of oxidation states.

The term "ideal gas" refers to a gas for which certain assumptions have been made. Which of the following is such an assumption? A) the law PV = nRT^2 is strictly obeyed B) intermolecular molecular forces are infinitely large C) individual molecular volume and intermolecular forces are negligible D) one gram-mole occupies a volume of 22.4L at 25 degree C and one atmosphere pressure

C; This is a General Chemistry question that falls under the content category "Importance of fluids for the circulation of blood, gas movement, and gas exchange." The answer to this question is C since a property of an "ideal" gas is that it is composed of particles that have negligible volume and do not exert intermolecular forces. It is a Knowledge of Scientific Concepts and Principles question since you are asked to recognize a component of the Ideal Gas law.

Which single bond present in nitroglycerin is the LEAST polar? A) C-H B) C-O C) C-C D) O-N

C; This is a General Chemistry question that falls under the content category "Nature of molecules and intermolecular interactions." The answer to this question is C because bonds between identical atoms are likely to be the least polar. This is true in this case since each C atom is bonded to one nitro group only. It is a Scientific Reasoning and Problem Solving question because you must use the theories of covalent bonding and electronegativity to predict the relative polarity of bonds within a molecule.

Which of the following will decrease the percentage ionization of 1.0 M acetic acid, CH3CO2H (aq)? A) chlorinating the CH3 group B) diluting the solution C) adding concentrated HCl (aq) D) adding a drop of basic indicator

C; This is a General Chemistry question that falls under the content category "Unique nature of water and its solutions." The answer to this question is C because HCl is a strong acid that will increase the amount of H+ in solution and thus decrease the percentage of CH3CO2H that ionizes. It is a Scientific Reasoning and Problem Solving question because you are asked to reason using a scientific principle (Le Châtelier's principle) to identify that adding a strong acid to a solution of weak acid will decrease the amount of ionization of the latter.

The pH of a 1 L phosphate buffer solution was measured as 7.6, but the experimental procedure calls for a pH 7.2 buffer. Which method will adjust the solution to the proper pH? (Note: The pKa values for phosphoric acid are 2.2, 7.2, and 12.3.) A) add enough 1 M Na2HPO3 to increase the phosphate anion concentration ten-fold. B) add 1 M NaOH to neutralize a portion of the hydronium ions found in the solution. C) alter the ratio of monosodium/disodium phosphate added to favor the monosodium species. D) add 100 mL distilled, deionized water to dilute the basicity of the buffer.

C; This is a General Chemistry question that falls under the content category "Unique nature of water and its solutions." The answer to this question is C because, in order to lower the pH of a buffer, the proportion of acidic buffer component must be increased. Adding strong base, diluting with water, or adding a different basic salt will not lower the pH. It is a Reasoning about the Design and Execution of Research question because you reason about the appropriateness of a specific experimental change in order to conduct research in the natural sciences.

What is the concentration of Ca^2+(aq) in a saturated solution of CaCO3? (Note: The solubility product constant Ksp for CaCO3 is 4.9 × 10^-9.) A) 2.4 × 10^-4 M B) 4.9 × 10^-5 M C) 7.0 × 10^-5 M D) 4.9 × 10^-9 M

C; This is a General Chemistry question that falls under the content category "Unique nature of water and its solutions." The answer to this question is C. The solubility product constant expression for CaCO3 is Ksp = [Ca^2+][CO3^2-]. Since equal quantities of Ca^2+(aq) and CO3^2-(aq) are produced when CaCO3 dissolves, this expression reduces to 4.9 × 10^-9 = x^2, or 49 × 10^-10 = x^2. This can be solved directly by taking the square root of each side. It is a Scientific Reasoning and Problem Solving question because you determine and use scientific formulas to solve a problem.

*From passage: "The best-performing PANI had a maximum conductivity of 5.0x10^-3 (Ω•cm)^-1" What is the resistivity of the best-performing PANI described in the passage? A) 0.002 Ω•cm B) 50 Ω•cm C) 200 Ω•cm D) 500 Ω•cm

C; This is a Physics question that falls under the content category "Electrochemistry and electrical circuits and their elements." The answer to this question is C because resistivity is the inverse of the conductivity, which is 1/5.0 × 10-3 (Ω∙cm)-1 = 200 Ω∙cm. It is a Scientific Reasoning and Problem Solving question because it asks you to determine and use scientific formulas to solve problems.

A person, whose eye has a lens-to-retina distance of 2.0 cm, can only clearly see objects that are closer than 1.0 m away. What is the strength S of the person's eye lens? (Note: Use the thin lens formula 1/O + 1/I = S.) A) -50 D B) -10 D C) 51 D D) 55 D

C; This is a Physics question that falls under the content category "How light and sound interact with matter." The answer to this question is C because the strength of the eye lens is equal to the inverse of the focal length of the eye lens. Its numerical value is given by ​​(1/1m)+(1/0.02 m)=1 D + 50 D=51 D. It is a Knowledge of Scientific Concepts and Principles question because you are asked to use mathematical equations to solve problems.

A tall tube is evacuated, and its stopcock closed. The open end of the tube is immersed into a container of water (density 10^3 kg/m^3) that is open to the atmosphere (pressure 10^5 N/m^2). When the stopcock is opened, how far up the tube will the water rise? A) 1 m B) 5 m C) 10 m D) 20 m

C; This is a Physics question that falls under the content category "Importance of fluids for the circulation of blood, gas movement, and gas exchange." The answer to this question is C because the water will rise to a height such that the weight (mass multiplied by gravitational acceleration) of the water column equals the atmospheric pressure multiplied by the tube cross-sectional area A. Because mass is density times volume, it follows that 10^3 kg/m^3 × h × A× 10 m/s^2 = 10^5 N/m^2 × A, where h is the height sought. Solving for h yields h = 10^5N/m^2/(10^4 N/m^3) = 10 m. It is a Scientific Reasoning and Problem Solving question because it asks you to determine and use scientific formulas to solve problems.

*See passage for #9 on AAMC Test 1* What is the product of the reaction of Compound 1 (shown below) with HBr by the pathway shown in Figure 3? A) (R)-1-bromo-1-deuteriohexane B) (S)-1-bromo-1-deuteriohexane C) (S)-1-bromo-1-deuteriopentane D) (R)-1-bromo-1-deuteriopentane

C; This is an Organic Chemistry question that falls under the content category "Nature of molecules and intermolecular interactions." The answer to this question is C because the incoming nucleophile displaces the leaving group form the opposite side of the reacting center during an SN2 reaction. It is a Scientific Reasoning and Problem Solving question because you are asked to bring together theory and observations to draw a conclusion.

*See passage for #4 on AAMC Test 2* What is the total number of sp2-hybridized carbon atoms present in the fluorophore used in the experiments? A) 4 B) 6 C) 9 D) 10

C; This is an Organic Chemistry question that falls under the content category "Nature of molecules and intermolecular interactions." The answer to this question is C. The fluorophore used in the experiments was AMC. There are 10 carbon atoms in AMC. Nine of these carbon atoms are AX3 systems that possess exactly one doubly bonded atom and are therefore sp2-hybridized. It is a Scientific Reasoning and Problem Solving question because you must use valence bond theory and the observed structure of a molecule to draw a conclusion about the number of sp2-hybridized carbon atoms present.

*See passage for #4 on AAMC Test 1* Which of the following statements does NOT correctly describe the dehydration of malic acid to fumaric acid and maleic acid? A) the reaction occurs most readily with tertiary alcohols B) the reaction involves the loss of a water molecule C) the reaction has a carbocation intermediate D) the reaction is stereospecific

C; This is an Organic Chemistry question that falls under the content category "Nature of molecules and intermolecular interactions." The answer to this question is D because the fact that both fumaric and maleic acid are produced means that the dehydration of malic acid is NOT stereospecific. It is a Data-based and Statistical Reasoning question because you are asked to analyze and interpret data (the outcome of a particular organic chemical reaction) in order to draw a conclusion.

*See passage for #17 on AAMC Test 2* What is the net charge of sT-loop at pH 7.2? A) -2 B) -1 C) 0 D) +1

C; This is an Organic Chemistry question that falls under the content category "Structure, function, and reactivity of biologically-relevant molecules." The answer to this question is C because at pH 7.2, the N-terminus will be positively charged and the C-terminus will be negatively charged. In addition, the lysine side chain will carry one positive charge and the glutamic acid side chain will carry one negative charge. It is a Scientific Reasoning and Problem Solving question because you must calculate a value based on the scientific principle of amino acid charge to solve the problem.

A circuit consists of a 12 V battery and two resistors of resistance 3 Ω and 4 Ω in parallel with the battery. The ratio of the current in the 3 Ω resistor to that in the 4 Ω resistor is: A) 1:1 B) 3:4 C) 4:3 D) 12:7

C; To correctly answer this question, one must know the equation for determining current: I = V/R where I = current, V = voltage and R = resistance. The current in the 3 Ω resistor is thus 12 V/3 Ω = 4 A. The current in the 4 Ω resistor is 12 V/4 Ω = 3 A. Thus, the ratio of the current in the 3-ohm resistor to that in the 4-ohm resistor is: 4 A/3 A = 4/3, choice (C) .

A long straight wire carries a current directed towards the top of the page. A proton moves with velocity 2 × 10^4 cm/s towards the bottom of the page and to the left of the wire. The magnetic force on the proton is: A) out of the page B) into the page C) towards the left D) towards the right

C; To figure out the direction of the magnetic field, use the right hand rule for the long straight wire. The right thumb points in the direction of the current, and the remaining right fingers curl around the wire in the direction of the field lines. So, if we point our thumb towards the top of the page, we see that our fingers (and thus the field) comes out of the page to the left of the wire, and goes into the page to the right of the wire. To find the magnetic force on the proton, we must use the right-hand rule for the magnetic force on a moving charge. This time, let the thumb of the right hand point in the direction of the proton's movement. Thus, it will point down. The remaining fingers of the right hand should point in the direction of the magnetic field, which is out of the page (remember, we determined above that the magnetic field comes out of the page to the left of the wire). The palm of the right hand should now face the direction of the force — in our case, towards the left, choice (C) .

What is the electric potential energy of a +2 C change held at a distance (from a stationary charge) with a potential of 30 volts? A) 15 J B) 30 J C) 60 J D) cannot be determined from the information given

C; Voltage is defined as V = kQ/r and electrical potential energy is U = kQq/r. Therefore electrical potential energy can be rewritten as U = Vq. Since the charge is +2 C and the voltage is 30 V the electrical potential energy should be (C) , 60 J.

Electrons in the 1s subshell of which of the following elements experience the greatest effective nuclear charge? a) H b) Na c) Cl d) O

C; Zeff increases from left to right across the periodic table because of the increase in positive charge in the nucleus. Because the question asks about 1s electrons, shielding effects can be ignored making the number of protons the most important factor. Thus, (C) is correct.

Which of the following is true of a galvanic cell? A) oxidation occurs at the cathode, reduction occurs at the anode, and the reaction is nonspontaneous B) oxidation occurs at the anode, reduction occurs at the cathode, and the reaction is nonspontaneous C) oxidation occurs at the cathode, reduction occurs at the cathode, and the reaction is spontaneous D) oxidation occurs at the anode, reduction occurs at the cathode, and the reaction is spontaneous

D; A galvanic cell is defined as a positive EMF (Ecell > 0), which means that the reaction is spontaneous since ΔG =-nFE . Since Ecell>0, ΔG<0, so the reaction is spontaneous, ruling out choices (A) and (B) . Using the mnemonic RED CAT and AN OX, reduction always occurs at the cathode for every type of electrochemical cell and oxidation always occurs at the anode. Therefore, choice (D) is correct.

*See passage for #3 on Kaplan Test 2 (short)* Two soil samples of unequal mass, both contaminated with radioactivity, are examined separately with a Geiger counter. Which of the following statements must be true? A) the sample with the shorter half-life will produce a higher frequency of clicks B) the sample with the longer half-life will produce a higher frequency of clicks C) the sample with the shorter half-life will generate a larger current D) the half-life cannot be determined from the click frequency alone

D; As explained in Paragraph 3, the Geiger counter's clicks are proportional to the decay in the sample, and therefore are proportional to the half-life of the radionuclide. While the half-life can serve as a relative measure of the frequency of decay between two samples of equal mass, without knowing the masses of the two samples, the absolute frequency of decays cannot be determined. For example, a huge sample with a long half-life may produce more decays (and therefore more clicks) than a small sample with an extremely short half-life. Therefore, choice (D) is correct.

A projectile of mass 20kg is shot at an angle of 60° to the horizontal at a speed of 100 m/s. What is the total mechanical energy of the projectile at its highest point? A) 0 J B) 2.5x10^4 J C) 7.5x10^4 J D) 1x10^5 J

D; At the highest point, the projectile will possess two types of mechanical energy: potential energy, because of its height and kinetic energy, because of its speed. The total mechanical energy of the projectile is equal to the sum of the kinetic and potential energies. This question also doesn't indicate any dissipative forces acting, which means that the total energy at the highest point will be equal to the energy at the beginning when all energy is kinetic. The equation for kinetic energy KE = 1/2mv^2 = 1/2(20)(100) 2 = (10)(10000) = 1 x 10^5 , or choice (D) .

How do beta and gamma radiation differ? (from passage: "beta decay emits an electron, while gamma decay is the emission of a high energy photon.") A) gamma rays can penetrate the Geiger counter cylinder, while beta rays cannot B) beta particles and gamma rays will be deflected in opposite directions by a magnetic field C) beta particles and gamma rays will be deflected in opposite directions by an electric field D) beta particles will be deflected in a magnetic field while gamma rays will not

D; Beta particles have a negative charge. They are electrons, as stated in Paragraph 1. Beta particles will be deflected by a magnetic field, because they are charged. Gamma rays are high energy photons lacking any charge. They will not be deflected in a magnetic field or an electric field precisely because they are uncharged. Therefore, choice (D) is the correct answer. Choice (A) is incorrect because beta rays can penetrate a Geiger counter cylinder. Furthermore, the passage does not discuss the entry of particles into the Geiger counter other than to say that it happens.

Which of the following lists of values, obtained from bomb calorimetry, constitutes sufficient information to determine the bond energy of activated glucosamine? A) Mass of reactants, heat of formation of reactants, temperature of products B) temperature of reactants, mass of calorimeter and specific heat of calorimeter C) mass of water in calorimeter and specific heat of water D) none of the above is sufficient to determine the bond energy

D; Bomb calorimetry is a very inefficient way to determine the bond enthalpies within a compound when not starting from a standard state. In order to do so, we would need all of the information provided above, plus the heat of formation of the products and background information on bond energies contained within either the reactants or products. Choice (D) is correct.

Which of the processes below is accompanied by the greatest increase in entropy? A) freezing B) condensation C) evaporation D) sublimation

D; During phase changes, entropy increases when substances change phase from a solid to a liquid, a liquid to a gas, or a solid to gas. Generally, gases have the greatest amount of entropy, while solids have the smallest. Sublimation, choice (D) , is the change in phase from a solid to a gas, so it will have the greatest increase in entropy.

If the pH of blood were to increase to 7.6, what would be the likely outcome? A) an increase in carbonic anhydrase activity B) a decrease in carbonic anhydrase activity C) an increase in the rate of CO2 exhalation D) a decrease in the rate of CO 2 exhalation

D; If the pH of blood increases to 7.6, it becomes more alkaline, and the body will start working to bring back the pH to 7.4. Since it is stated in the second sentence of Paragraph 3 that blood pH can be adjusted rapidly by changes in the rate of CO2 exhalation, choices (A) and (B) can be eliminated. In order to bring the pH of blood back to its normal value of 7.4, it must become more acidic; it becomes more acidic by increasing the concentration of H+. From Le Châtelier's principle, the product concentration will increase if reactant is added to the system. Thus, if the concentration of carbon dioxide is allowed to increase, it will react to produce more H+, resulting in a lowering of the pH. The concentration of carbon dioxide will increase if it is not exhaled, so choice (D) is the correct response.

According to the following standard reduction potentials: Zn^2+ (aq) + 2 e- → Zn(s) E° = -0.76 V Ag+ (aq) + e - → Ag(s) E° = +0.8 V What is the cell potential of the reaction below? Zn(s) + 2Ag+ (aq) → Zn^2+ (aq) + 2Ag(s) A) -1.56 V B) -0.04 V C) +0.04 V D) +1.56 V

D; In this reaction, zinc solid is oxidized and the Ag+ is reduced. Therefore, the standard oxidation potential of Zn(s) is determined by reversing the sign of the standard reduction potential for Zn^2+ or E° = 0.76 V. To determine the overall E° for the cell, add the standard oxidation potential of Zn(s) to the standard reduction potential for Ag+. The sum will equal to (D), E° = 1.56 V (0.76 +0.8).

Which of the following would produce a ketone when it undergoes Jones oxidation? A) ethanol B) 1-propanol B) isobutanol D) 2-propanol

D; Jones oxidation oxidizes primary alcohols to carboxylic acids and secondary alcohols into ketones. The only secondary alcohol present is choice (D) , 2-propanol.

On a globular protein, what are the preferred locations for leucine and phenylalanine residues? A) Leu prefers the interior, while Phe prefers the exterior B) Leu prefers the exterior, while Phe prefers the interior C) Leu and Phe both prefer the exterior D) Leu and Phe both prefer the interior

D; Leu and Phe have hydrophobic side chains and thus prefer to be buried on the interior. Thus, choice (D) is correct.

Which of the following CANNOT be the mechanism for the rate-determining step of this reaction (2A+B <-> 2C+D)? A) a molecule of A collides with a molecule of B B) a molecule of A collides with another molecule of A C) a molecule of B collides with another molecule of B D) two molecules of A collide with a molecule of B

D; Paragraph 3 states that the slower step of the reaction mechanism follows second-order kinetics. The slowest step of a reaction mechanism is usually the rate-determining step, and a step that involves second-order kinetics must only involve two reactant molecules. Although it is often said that the kinetics of a reaction cannot be derived from its stoichiometry, this is true only of the overall reaction. When a complex reaction has been broken down into a series of elementary reactions, we can then derive the rate law from the slowest of the elementary reactions. again, in this case, we know that the rate-determining step must consist of an interaction between two molecules. This could be two molecules of A, two of B, or one each of A and B. Choice (D) involves three reactant molecules, and this therefore third-order, and incorrect.

A spring with a 5kg weight attached is in harmonic motion. As the mass passes through the equilibrium position (x0), the mass has 40J of kinetic energy. At x=20cm, the spring has 20J of kinetic energy. The spring constant for the spring is: A) 200 N/m B) 400 N/m C) 800 N/m D) 1000 N/m

D; Since spring forces are conservative, TE = KE + PE. Here, the TE is 40 J since x0 is where all of the energy is kinetic. This means that at any given point KE + PE = 40 J. At 20 cm, the spring has a KE of 20 J so PE must also be 20 J. Using the equation for potential energy of spring systems, PE = 1/2kx^2 , 20 J = 1/2(k)(0.2 m)^2 . Solving for k yields k = 1000 N/m or choice (D) .

*See passage for #40 on Kaplan Test #1* The activator molecule mentioned in Paragraph 1 contains: A) a deoxyribose and a purine base B) a deoxyribose and a pyrimidine base C) a ribose and a purine base D) a ribose and a pyrimidine base

D; The activator molecule mentioned in Paragraph 1 is UTP (uridine 5'-triphosphate). It is derived from the nitrogenous base uracil. Uracil is found in RNA (which contains ribose), not DNA (which contains deoxyribose), so the choices can be quickly narrowed to choices (C) and (D). To narrow down to a single, correct answer choices, remember that uracil is a pyrimidine. Hence, choice (D) is the correct answer choice.

The mass number of a neutral element corresponds to the number of: A) protons B) protons plus electrons C) neutrons D) protons plus neutrons

D; The mass number of an atom is the sum of the neutrons plus protons in an atom. Thus, the number of neutrons can be determined by subtracting the atomic number of an atom from its mass number, or choice (D) .

*See passage for #43 on AAMC Test 1* Compared to normal glycogen, the amount of what type of bond is decreased in Lafora bodies? A) Phosphomonoester B) Phosphodiester C) alpha-1,4-Glycosidic bond D) alpha-1,6-Glycosidic bond

D; This Biochemistry question falls under the content category "Structure, function, and reactivity of biologically-relevant molecules." The answer D because it is stated in the passage that the lack of branching is a characteristic of Lafora bodies. Branching occurs through an α-1,6-glycosidic bond in glycogen. This is a Knowledge of Scientific Concepts and Principles question because you are asked to recall the properties of glycogen branching.

Which experimental condition is NOT necessary to achieve reliable data for Michaelis-Menten enzyme kinetics? A) Initial velocity is measured under steady state conditions. B) Solution pH remains constant at all substrate concentrations. C) The concentration of enzyme is lower than that of substrate. D) The reaction is allowed to reach equilibrium before measurements are taken.

D; This is a Biochemistry question that falls under the content category "Principles of chemical thermodynamics and kinetics." The answer to this question is D because once the reaction reaches equilibrium, measurement of Vo will be impossible and the kinetic data will look the same regardless of substrate concentration. Hence, response D is not necessary (nor desirable) to achieve reliable data for Michaelis−Menten enzyme kinetics. In contrast, Distractors A−C are essential to obtain reliable Vo versus substrate concentration data to calculate KM and Vmax using Michaelis−Menten enzyme kinetics. It is a Reasoning about the Design and Execution of Research question because the question probes the experimental conditions required to obtain reliable data.

*See passage for # 35 on AAMC Test 1 (Specifically Table 1)* Compared to the wild-type LipA, what is the change in net charge in variant XI at pH 7? A) +4 B) +3 C) -3 D) -4

D; This is a Biochemistry question that falls under the content category "Structure, function, and reactivity of biologically-relevant molecules." The answer is D because the amino acid substitutions replace amino acids with charges of +1, +1, 0, 0, and 0 with ones that have charges of 0, −1, −1, 0, and 0 so the net charge goes from +2 to −2. This is a Scientific Reasoning and Problem Solving question because you are asked to determine the necessary calculation to solve for net charge.

*See passage for #41 on AAMC Test 3* What is the correct expression for the ΔG′° for the transition observed in the experiments described in the passage? A) ΔG′° = -RTe([native]/[unfolded]) B) ΔG′° = -RTe([unfolded]/[native]) C) ΔG′° = -RTln([native]/[unfolded]) D) ΔG′° = -RTln([unfolded]/[native])

D; This is a Biochemistry question that falls under the content category "Structure, function, and reactivity of biologically-relevant molecules." The answer to this question is D because the equilibrium constant for DNA unfolding is Keq=[unfolded]/[native] as the unfolded DNA is considered to be a product. Thus, ΔG′° = −RTlnKeq = −RTln([unfolded]/[native]). It is a Scientific Reasoning and Problem Solving question because it requires a scientific equation and its application to experimental data.

*See passage for #19 on AAMC Test 2* When used in place of spHM, which peptide would be most likely to achieve the same experimental results? A) FLGFAY B) FLGFQY C) FLGFGY D) FLGFEY

D; This is a Biochemistry question that falls under the content category "Structure, function, and reactivity of biologically-relevant molecules." The answer to this question is D because the phosphorylated threonine would most likely be mimicked by glutamic acid in terms of size and charge. It is a Reasoning about the Design and Execution of Research question because you must reason about how a change in the experiment design would affect the outcome of the experiment.

*See passage for #13 on AAMC Test 2* The side chain of tryptophan will give rise to the largest CD signal in the near UV region when: A) present as a free amino acid B) part of an alpha-helix C) part of a beta-sheet D) part of a fully folded protein

D; This is a Biochemistry question that falls under the content category "Structure, function, and reactivity of biologically-relevant molecules." The answer to this question is D because tryptophan has an aromatic side chain that will give rise to a significant CD signal in the near UV region if it is found in a fully folded protein. It is a Scientific Reasoning and Problem Solving question because you must bring together theory, observations, and evidence to draw a conclusion.

*See passage for #23 on AAMC Test 1* Which amino acid residues were incorporated into Compound 1 to promote the adhesion of cells on the scaffold surfaces? A) Arg and Gly B) Cys and Gly C) Cys ad Asp D) Asp and Arg

D; This is a Biochemistry question that falls under the content category "Structure, function, and reactivity of biologically-relevant molecules." The answer to this question is D. The residues that were engineered into the peptide for cell adhesion are arginine and aspartate as can be reasoned based on the structure of the peptide provided, and the description of the roles of the various residues provided in the passage. It is a Scientific Reasoning and Problem Solving question because you are asked to reason using scientific principles and models.

*See passage for #48 on AAMC Test 1* Which of the following is the overall reaction for the decomposition of H2O2 that is shown in reactions 3 and 4? A) 2H2O2 (aq) + IO- (aq) -> 2H2O (l) + I- (aq) + O2 (g) B) 2H2O2 (aq) + 2I- (aq) -> 2H2O (l) + I2 (g) + O2 (g) C) H2O2 (aq) -> H2O (l) + O2 (g) D) 2H2O2 (aq) -> 2H2O (l) + O2 (g)

D; This is a General Chemistry question that falls under the content category "Atoms, nuclear decay, electronic structure, and atomic chemical behavior." The answer to this question is D because this response depicts the net sum of reactions 3 and 4. It is a Knowledge of Scientific Concepts and Principles question because you are asked to recognize the scientific principle of chemical balance.

*See passage for #55 on AAMC Test 1* A circuit similar to that in Figure 2 is set up in which X=Pb, X^n+ = PB^2+, Y=Cu, and Y^m+ = Cu^2+. Will an oxidation-reduction reaction occur spontaneously? A) no; E cell = -0.212 B) no; E cell = +0.212 C) yes; E cell = -0.466 D) yes; E cell = +0.466

D; This is a General Chemistry question that falls under the content category "Electrochemistry and electrical circuits and their elements." The answer to this question is D. A spontaneous reaction occurs when E° is greater than zero, and this occurs if the oxidation of Pb(s) is combined with the reduction of Cu2+(aq), resulting in a net E° of +0.466 V. It is a Scientific Reasoning and Problem Solving question because you are asked to determine and use a scientific formula to solve a problem after recognizing the relationship between E° and spontaneity.

*See passage for #50 on AAMC Test 2* At STP, the volume of N2(g) produced by the complete decomposition of 1 mole of nitroglycerin would be closest to which of the following? A) 5 L B) 10 L C) 20 L D) 30 L

D; This is a General Chemistry question that falls under the content category "Importance of fluids for the circulation of blood, gas movement, and gas exchange." The answer to this question is D. Based on the balanced equation provided, 4 moles of nitroglycerin produce 6 moles of N2(g). Therefore, 1 mole of nitroglycerin will produce 1.5 moles of N2(g). At STP 1.5 moles of N2(g) will occupy 33.6 L since the molar volume of an ideal gas at STP is 22.4 L/mol. It is a Scientific Reasoning and Problem Solving question because you determine and use scientific formulas to solve a problem.

Which of the following types of orbitals of the central atom are involved in bonding in octahedral compounds? A) sp B) sp^3 C) p D) d^2 sp^3

D; This is a General Chemistry question that falls under the content category "Nature of molecules and intermolecular interactions." The answer to this question is D because octahedral compounds have six σ bonds and no lone pairs. According to valence bond theory, the central atom requires the hybridization of six atomic orbitals, d2sp3. It is a Knowledge of Scientific Concepts and Principles question since you are asked to recognize the relationship between the closely related concepts of chemical bonding and hybridization.

Which of the following best describes the bonds between Cu^2+ and the nitrogen atoms of the ammonia molecules in [Cu(NH3)4]^2+? A) ionic B) covalent C) coordinate ionic D) coordinate covalent

D; This is a General Chemistry question that falls under the content category "Nature of molecules and intermolecular interactions." The answer to this question is D because the Lewis acid-base interaction between a metal cation and an electron pair donor is known as a coordinate covalent bond. It is a Knowledge of Scientific Concepts and Principles question because you must recognize a correct scientific principle.

The energy of activation for the reaction described in the passage is given by the energy of: A) the reactants minus the energy of the products B) the products minus the energy of the reactants C) the activated complex minus the energy of the products D) the activated complex minus the energy of the reactants

D; This is a General Chemistry question that falls under the content category "Principles of chemical thermodynamics and kinetics." The answer to this question is D because the activation energy for a reaction represents the minimum energy barrier necessary to be overcome by the reactants on the path to products. It is a Knowledge of Scientific Concepts and Principles question because you must identify the relationship among the closely related concepts of activation energy for a reaction, energies of the reactants, and energy of the activated complex.

*See passage for #49 on AAMC Test 2* Based on the passage, the magnitude of ΔH° (in kJ) for the decomposition of 2 moles of nitroglycerin at 25°C is closest to which of the following? A) 500 B) 1000 C) 2000 D) 3000

D; This is a General Chemistry question that falls under the content category "Principles of chemical thermodynamics and kinetics." The answer to this question is D. The value of ΔH° can be calculated using the data provided in Table 1 and applying Hess's Law. Two moles of nitroglycerin produce 6 moles of CO2(g) and 5 moles of H2O(g). The value of ΔH° for this amount of nitroglycerin combusted is 2(364.0) - 6(393.5) - 5(241.8) = -2842 kJ/mol. It is a Data-based and Statistical Reasoning question because you must consult and use data from a table to perform a calculation that answers a research question.

What volume of a 0.120M CaI2 solution would contain 0.078 mol of the solute? A) 35.0 mL B) 65.0 mL C) 350 mL D) 650 mL

D; This is a General Chemistry question that falls under the content category "Unique nature of water and its solutions." The answer to this question is D. In order to obtain the volume of solution necessary to provide a given amount of solute in moles, one needs to take the number of moles and divide by the solution concentration in molarity (0.078 mol/0.120M). It is a Scientific Reasoning and Problem Solving question because you are asked to determine and then use a formula to solve a scientific problem.

*See passage for #56 on AAMC Test 1* A circuit similar to that in Figure 2 is set up. It has a potential of 2.0 V. Assume that the resistance in the circuit is negligible compared to that of the lightbulb. Approximately how much current passes through the lightbulb? A) 0.5 A B) 1.0 A C) 2.0 A D) 4.0 A

D; This is a Physics question that falls under the content category "Electrochemistry and electrical circuits and their elements." The answer to this question is D because according to Ohm's law, current is equal to voltage divided by resistance, hence 2.0 V/0.5 Ω = 4.0 A. It is a Scientific Reasoning and Problem Solving question because you are asked to determine and use scientific formulas to solve problems.

What is the energy of the photons emitted by the LED at a frequency of 610 THz? (Note: h= 6.6x10^-34 Js) A) 9.2x10^-12 J B) 1.6x10^-16 J C) 1.1x10^-18 J D) 4.0x10^-19 J

D; This is a Physics question that falls under the content category "How light and sound interact with matter." The answer to this question is D because the energy of a photon of frequency 610 THz is equal to 6.6 x 10^-34 J•s x 610 x 10^12Hz = 4 x 10^-19 J. It is a Scientific Reasoning and Problem Solving question because you are asked to determine and use scientific formulas to solve problems.

*See passage for # 16 on AAMC Test 1 (don't really need to for the answer though)* The intensity of the radiation emitted by the oxygen sensor is directly proportional to the: A) propagation speed of the radiation B) wavelength of the radiation C) polarization of photons emitted D) number of photons emitted

D; This is a Physics question that falls under the content category "How light and sound interact with matter." The answer to this question is D because the energy of electromagnetic radiation is directly proportional to the number of photons, and the intensity of electromagnetic radiation is defined as energy emitted per unit time. Thus, intensity is directly proportional to the number of photons emitted. It is a Knowledge of Scientific Concepts and Principles question because you are asked to identify a relationship between closely related concepts.

*See passage for #55 on AAMC Test 2* What kind of image is formed by the lenses of the glasses worn by a 68-year-old male who sees an object 2 m away? A) Real and enlarged B) Real and reduced C) Virtual and enlarged D) Virtual and reduced

D; This is a Physics question that falls under the content category "How light and sound interact with matter." The answer to this question is D because the lenses have a negative focal length which means they are diverging lenses. Such lenses form virtual and reduced images of objects situated at distances larger than the focal length. It is a Knowledge of Scientific Concepts and Principles question because the question requires identifying the relationships among closely related concepts.

*See passage for #54 on AAMC Test 2* What is the ratio of the minimum sound intensities heard by a 64-year-old male and a 74-year-old female? A) 20 B) 40 C) 50 D) 100

D; This is a Physics question that falls under the content category "How light and sound interact with matter." The answer to this question is D because the relative intensities of the two sound waves are 20 dB and 40 dB, respectively. The difference is 20 dB, meaning that the decimal log of the ratio of their intensities is 2, which means that the ratio of their intensities is 100. It is a Scientific Reasoning and Problem Solving question because the question requires determining and using scientific formulas to solve problems.

*From the passage: "A study was done on the tissues of rats treated with microbubbles burst by 2.3 MHz ultrasound. It was observed that the burst microbubbles made openings of ~2.5x10^4 μm^2 through the capillary walls." Given that the speed of sound in the rat tissues was 1500 m/s, the wavelength of the ultrasound wave used in the study was closest to: A) 34.5 mm B) 6.5 mm C) 1.5 mm D) 0.65 mm

D; This is a Physics question that falls under the content category "How light and sound interact with matter." The answer to this question is D because the wavelength is given by (1500 m/s)/(2.3 MHz) = 0.65 mm. It is a Scientific Reasoning and Problem Solving question because it asks you to determine and use scientific formulas to solve problems.

*See passage for #21 on AAMC Test 1* What is the work generated by a healthy adult who circulates 9L of blood through the brachial artery in 10 min? A) 2 kJ B) 12 kJ C) 20 kJ D) 120 kJ

D; This is a Physics question that falls under the content category "Translational motion, forces, work, energy, and equilibrium in living systems." The answer to this question is D because a flow of 9 liters in 10 minutes means a flow rate of 900 mL/min, and according to the graph, it corresponds to a power of 200 W. The work is then 200 W x 600 s = 120 kJ. It is a Data-based and Statistical Reasoning question because you are asked to use, analyze, and interpret data in a graph.

A person is sitting in a chair as shown. (*image of person sitting straight up with their torso in a 90 degree angle with their legs and their knees bent at a 90 degree angle with their feet planted on the ground) Why must the person either lean forward or slide their feet under the chair in order to stand up? A) to increase the force required to stand up B) to use the friction with the ground C) to reduce the energy required to stand up D) to keep the body in equilibrium while rising

D; This is a Physics question that falls under the content category "Translational motion, forces, work, energy, and equilibrium in living systems." The answer to this question is D because as the person is attempting to stand, the only support comes from the feet on the ground. The person is in equilibrium only when the center of mass is directly above their feet. Otherwise, if the person did not lean forward or slide the feet under the chair, the person would fall backward due to the large torque created by the combination of the weight of the body (applied at the person's center of mass) and the distance along the horizontal between the center of mass and the support point. It is a Knowledge of Scientific Concepts and Principles question because the question requires identifying examples of observations that illustrate scientific principles.

*See passage and answer choices for #8 on AAMC Test 3* If the generator-brake system alone were engaged when the railcar was moving, which of the following graphs would most accurately represent the subsequent speed of the railcar? A) *see graph* B) *see graph* C) *see graph* D) *see graph*

D; This is a Physics question that falls under the content category "Translational motion, forces, work, energy, and equilibrium in living systems." The answer to this question is D because the speed of the car decreases in time such that the decelerating force declines linearly with speed, according to the passage. Because the decelerating force is directly proportional to the deceleration acceleration, it follows that the decelerating acceleration itself declines linearly with speed. Finally, because the decelerating acceleration is rate of change of velocity, or the slope of the velocity vs. time graph, then the slope of the graph must decrease in time. It is a Data-based and Statistical Reasoning question because it asks you to evaluate whether representations make sense for particular scientific observations and data.

The relative thermodynamic stability of isomeric organic compounds can be inferred from which of the following types of experimental data? A) Boiling points B) UV-visible absorption spectra C) Mass spectroscopic fragmentation patterns D) Heats of combustion

D; This is an Organic Chemistry question that falls under the content category "Nature of molecules and intermolecular interactions." The answer to this question is D because the relative thermodynamic stability of isomers can be determined based on the amount of heat produced when the compounds are combusted; less heat, greater stability. It is a Reasoning about the Design and Execution of Research question because one must choose the appropriate experimental data to answer the question.

*See passage for #6 on AAMC Test 1* If 2-pentanol replaces 1-pentanol in the reaction shown in Figure 3, the rate of substitution is less because: A) the C-O bond in 2-pentanol is stronger than the C-O bond in 1-pentanol B) there is a competing elimination reaction that slows the rate of substitution C) there is more steric hindrance at the oxygen atom in 2-pentanol than in 1-pentanol, making protonation less likely D) there is more steric hindrance at the 2- position of 2-pentanol than at the 1- position of 1-pentanol

D; This is an Organic Chemistry question that falls under the content category "Structure, function, and reactivity of biologically-relevant molecules." The answer to this question is D because the rate of substitution of protonated alcohols is subject to steric hindrance. This inhibits the ability of nucleophiles to collide with the reacting electrophilic center and slows the rate of reaction. It is a Scientific Reasoning and Problem Solving question because you are asked to evaluate an argument about causes and consequences (the reason a reaction proceeds more slowly when a different reactant is used).

Which of the following is a true statement about phospholipids and triglycerides? A) all triglycerides, but not all phospholipids, have saturated fatty acid tails B) all phospholipids, but not all triglycerides, have three ester moieties C) all phospholipids, but not all triglycerides, have three sp^2 - hybridized carbons D) all triglycerides, but not all phospholipids, have a glycerol backbone

D; Triglycerides (or triacylglycerols) are triesters derived from glycerol and three fatty acids (of varying degrees of saturation). Thus, choices A), (B), and (C) can be eliminated. Phospholipids are similar to triglycerides, but their backbone may or may not be glycerol (among other differences). choice (D) is the correct answer.

DD-transpeptidase is a bacterial enzyme that cross-links peptidoglycan chains to form rigid cell walls. Threonine is often found in these interbridges. Which of the following is also a likely component? A) Histidine B) Isoleucine C) Phenylalanine D) Serine

D; the question is essentially asking for the amino acid most similar to threonine. Serine, choice (D) is the most similar.


Kaugnay na mga set ng pag-aaral

Prospects and challenges for a sustainable foodsystem LV0103

View Set

Personal Finance Final multiple choice/matching/t or f

View Set

Foundations - Google Digital Marketing & Ecommerce

View Set

Medicare Agent & Broker Certification

View Set

Module 39: Basic Concepts of Psychological Disorders and Mood Disorders

View Set

lexemas y morfemas; prefijos y sufijos

View Set

Appellate Terms - Appellate Courts Only

View Set

IB Bio HL Test: Human Health & Physiology III

View Set